Site Loader

Содержание

Элементы цепи синусоидального тока. Векторные диаграммы и комплексные соотношения для них. (Лекция N 4)

1. Резистор

Идеальный резистивный элемент не обладает ни индуктивностью, ни емкостью. Если к нему приложить синусоидальное напряжение (см. рис. 1), то ток i через него будет равен

. (1)

Соотношение (1) показывает, что ток имеет ту же начальную фазу, что и напряжение. Таким образом, если на входе двухлучевого осциллографа подать сигналы u и i, то соответствующие им синусоиды на его экране будут проходить (см. рис. 2) через нуль одновременно, т.е. на резисторе напряжение и ток совпадают по фазе.

Из (1) вытекает:

;

.

 

Переходя от синусоидальных функций напряжения и тока к соответствующим им комплексам:

;

,

— разделим первый из них на второй:

или

. (2)

Полученный результат показывает, что отношение двух комплексов есть вещественная константа. Следовательно, соответствующие им векторы напряжения и тока (см. рис. 3) совпадают по направлению.

2. Конденсатор

Идеальный емкостный элемент не обладает ни активным сопротивлением (проводимостью), ни индуктивностью. Если к нему приложить синусоидальное напряжение (см. рис. 4), то ток

i через него будет равен

. (3)

Полученный результат показывает, что напряжение на конденсаторе отстает по фазе от тока на /2. Таким образом, если на входы двухлучевого осциллографа подать сигналы u и i, то на его экране будет иметь место картинка, соответствующая рис. 5.

Из (3) вытекает:

;

 

.

 

Введенный параметр называют реактивным емкостным сопротивлением конденсатора. Как и резистивное сопротивление, имеет размерность Ом. Однако в отличие от R данный параметр является функцией частоты, что иллюстрирует рис. 6. Из рис. 6 вытекает, что при конденсатор представляет разрыв для тока, а при .

Переходя от синусоидальных функций напряжения и тока к соответствующим им комплексам:

;

,

— разделим первый из них на второй:

или

. (4)

В последнем соотношении — комплексное сопротивление конденсатора. Умножение на соответствует повороту вектора на угол по часовой стрелке. Следовательно, уравнению (4) соответствует векторная диаграмма, представленная на рис. 7.

3. Катушка индуктивности

Идеальный индуктивный элемент не обладает ни активным сопротивлением, ни емкостью. Пусть протекающий через него ток (см. рис. 8) определяется выражением

. Тогда для напряжения на зажимах катушки индуктивности можно записать

. (5)

Полученный результат показывает, что напряжение на катушке индуктивности опережает по фазе ток на /2. Таким образом, если на входы двухлучевого осциллографа подать сигналы u и i, то на его экране (идеальный индуктивный элемент) будет иметь место картинка, соответствующая рис.

9.

Из (5) вытекает:


.

Введенный параметр называют реактивным индуктивным сопротивлением катушки; его размерность – Ом. Как и у емкостного элемента этот параметр является функцией частоты. Однако в данном случае эта зависимость имеет линейный характер, что иллюстрирует рис. 10. Из рис. 10 вытекает, что при катушка индуктивности не оказывает сопротивления протекающему через него току, и при .

Переходя от синусоидальных функций напряжения и тока к соответствующим комплексам:

;

,

разделим первый из них на второй:

или

. (6)

В полученном соотношении — комплексное

сопротивление катушки индуктивности. Умножение на соответствует повороту вектора на угол против часовой стрелки. Следовательно, уравнению (6) соответствует векторная диаграмма, представленная на рис. 11

 

4. Последовательное соединение резистивного и индуктивного элементов

Пусть в ветви на рис. 12 . Тогда

где

, причем пределы изменения .

Уравнению (7) можно поставить в соответствие соотношение

,


которому, в свою очередь, соответствует векторная диаграмма на рис. 13. Векторы на рис. 13 образуют фигуру, называемую треугольником напряжений. Аналогично выражение

графически может быть представлено треугольником сопротивлений (см. рис. 14), который подобен треугольнику напряжений.

 

5. Последовательное соединение резистивного и емкостного элементов

Опуская промежуточные выкладки, с использованием соотношений (2) и (4) для ветви на рис. 15 можно записать

., (8)

где

, причем пределы изменения .


На основании уравнения (7) могут быть построены треугольники напряжений (см. рис. 16) и сопротивлений (см. рис. 17), которые являются подобными.

6. Параллельное соединение резистивного и емкостного элементов

Для цепи на рис. 18 имеют место соотношения:

;

, где [См] – активная проводимость;

, где [См] – реактивная проводимость конденсатора.

Векторная диаграмма токов для данной цепи, называемая треугольником токов, приведена на рис. 19. Ей соответствует уравнение в комплексной форме

,

где ;

— комплексная проводимость;

.

Треугольник проводимостей, подобный треугольнику токов, приведен на рис. 20.

Для комплексного сопротивления цепи на рис. 18 можно записать

.

Необходимо отметить, что полученный результат аналогичен известному из курса физики выражению для эквивалентного сопротивления двух параллельно соединенных резисторов.

7. Параллельное соединение резистивного и индуктивного элементов

Для цепи на рис. 21 можно записать

;

, где [См] – активная проводимость;

, где [См] – реактивная проводимость катушки индуктивности.

Векторной диаграмме токов (рис. 22) для данной цепи соответствует уравнение в комплексной форме

,

где ;

— комплексная проводимость;

.

Треугольник проводимостей, подобный треугольнику токов, приведен на рис. 23.


Выражение комплексного сопротивления цепи на рис. 21 имеет вид:

.

Литература

1.     Основы теории цепей: Учеб. для вузов /Г.В.Зевеке, П.А.Ионкин, А.В.Нетушил, С.В.Страхов. –5-е изд., перераб. –М.: Энергоатомиздат, 1989. -528с.

2.     Бессонов Л.А. Теоретические основы электротехники: Электрические цепи. Учеб. для студентов электротехнических, энергетических и приборостроительных специальностей вузов. –7-е изд., перераб. и доп. –М.: Высш. шк., 1978. –528с.

Контрольные вопросы и задачи

1.     В чем сущность реактивных сопротивлений?

2.     Какой из элементов: резистор, катушку индуктивности или конденсатор – можно использовать в качестве шунта для наблюдения за формой тока?

3.      Почему катушки индуктивности и конденсаторы не используются в цепях постоянного тока?

4.     В ветви на рис. 12 . Определить комплексное сопротивление ветви, если частота тока .
Ответ: .

5.     В ветви на рис. 15 . Определить комплексное сопротивление ветви, если частота тока .
Ответ: .

6.     В цепи на рис. 18 . Определить комплексные проводимость и сопротивление цепи для .
Ответ: ; .

7.     Протекающий через катушку индуктивности ток изменяется по закону А. Определить комплекс действующего значения напряжения на катушке.
Ответ: .

6.10. Мощность в цепи синусоидального тока

     Мгновенной мощностью называют произведение мгновенного напряжения на входе цепи на мгновенный ток.
     Пусть мгновенные напряжение и ток определяются по формулам:

     

     Тогда

          (6. 23)

     Среднее значение мгновенной мощности за период

     Из треугольника сопротивлений ,      а      .

     Получим еще одну формулу:

.

     Среднее арифметическое значение мощности за период называют активной мощностью и обозначают буквой P.
   Эта мощность измеряется в ваттах и характеризует необратимое преобразование электрической энергии в другой вид энергии, например, в тепловую, световую и механическую энергию.
     Возьмем реактивный элемент (индуктивность или емкость). Активная мощность в этом элементе , так как напряжение и ток в индуктивности или емкости различаются по фазе на 90o. В реактивных элементах отсутствуют необратимые потери электрической энергии, не происходит нагрева элементов.
   Происходит обратимый  процесс в  виде обмена электрической энергией между источником и приемником. Для качественной оценки интенсивности обмена энергией вводится понятие реактивной мощности Q.
     Преобразуем выражение (6.23):

     где — мгновенная мощность в активном сопротивлении;

      — мгновенная мощность в реактивном элементе (в индуктивности или в емкости).
   Максимальное или амплитудное значение мощности p2 называется реактивной мощностью

      ,

     где x — реактивное сопротивление (индуктивное или емкостное).
     Реактивная мощность, измеряемая в вольтамперах реактивных, расходуется на создание магнитного поля в индуктивности или электрического поля в емкости. Энергия, накопленная в емкости или в индуктивности, периодически возвращается источнику питания.
     Амплитудное значение суммарной мощности p = p1 + p2 называется полной мощностью.
   Полная  мощность,  измеряемая в вольтамперах, равна произведению действующих значений напряжения и тока:

      ,

     где z — полное сопротивление цепи.
   Полная мощность характеризует предельные возможности источника энергии. В электрической цепи можно использовать часть полной мощности

,

       где    — коэффициент мощности или «косинус «фи».

  Коэффициент  мощности  является одной из важнейших характеристик электротехнических устройств. Принимают специальные меры к увеличению коэффициента мощности.
      Возьмем треугольник сопротивлений и умножим его стороны на квадрат тока в цепи. Получим подобный треугольник мощностей (рис. 6.18).

     Из треугольника мощностей получим ряд формул:

,      ,

             Рис.6.18
                                                                ,      .
     При анализе электрических цепей символическим методом используют выражение комплексной мощности, равное произведению комплексного напряжения на сопряженный комплекс тока.
     Для цепи, имеющей индуктивный характер (R-L цепи)

,

       где   
      — комплекс напряжения;
      — комплекс тока;
      — сопряженный комплекс тока;
      — сдвиг по фазе между напряжением и током.
     , ток как в R-L цепи, напряжение опережает по фазе ток.

     Вещественной частью полной комплексной мощности является активная мощность.
     Мнимой частью комплексной мощности — реактивная мощность.
     Для цепи, имеющей емкостной характер (R-С цепи), . Ток опережает по фазе напряжение.

.

     Активная мощность всегда положительна. Реактивная мощность в цепи, имеющей индуктивный характер, — положительна, а в цепи с емкостным характером — отрицательна.

6.11. Баланс мощностей

     Для схемы на рис. 6.19 запишем уравнение по второму закону Кирхгофа. Умножим левую и правую части уравнения на сопряженный комплекс тока

       где    — результирующее реактивное сопротивление;
               I2— квадрат модуля тока.

     где    — полная комплексная, активная и реактивная мощности источника питания.

     где — активная и реактивная мощности, потребляемые элементами схемы.

     Получим уравнение

     .      (6.24)

Рис. 6.19

     Два комплексных числа равны, если равны по отдельности их вещественные и мнимые части, следовательно уравнение (6.24) распадается на два:

 .     (6.25)

    Полученные равенства выражают законы сохранения активных и реактивных мощностей.

6.12. Согласованный режим работы электрической цепи.


Согласование нагрузки с источником

     В схеме на рис. 6.20
      — полное, активное и реактивное сопротивления источника ЭДС,
      — полное, активное и реактивное сопротивления нагрузки.
   Активная мощность может выделяться только в активных сопротивлениях цепи переменного тока.
     Активная мощность, выделяемая в нагрузке,

.     (6.26)

     Активная мощность, развиваемая генератором

.
Коэффициент полезного действия для данной схемы:

                    .
                 Рис. 6.20

     Из формулы (6.26) видно, что выделяемая в нагрузке мощность будет максимальной, когда знаменатель минимален. Последнее имеет место при , т.е. при . Это означает, что реактивные сопротивления источника и нагрузки должны быть одинаковы по модулю и иметь разнородный характер. При индуктивном характере реактивного сопротивления источника реактивное сопротивление нагрузки должно быть емкостным и наоборот.

.     (6.27)

   Установим условие,  при котором  от источника к нагрузке будет передаваться наибольшая мощность.

.

     отсюда .

     От источника к нагрузке передается наибольшая мощность, когда

.      .     (6.28)

     Величина наибольшей мощности

.

   Режим передачи наибольшей мощности от источника к нагрузке называется согласованным режимом, а подбор сопротивлений согласно равенствам (6. 28) — согласованием нагрузки с источником.

     В согласованном режиме

.

     Половина мощности теряется внутри источника. Поэтому согласованный режим не используется в силовых энергетических цепях. Этот режим используют в информационных цепях, где мощности могут быть малыми, и решающими являются не соображения экономичности передачи сигнала, а максимальная мощность сигнала в нагрузке.

ответы по эл. технике

№1 Линейные цепи постоянного тока

1. Что такое электрический ток?

Направленное движение электрических зарядов, которое происходит под действием сил Эл. Поля. За положительное направление тока условно принято направление двитжения положительных зарядов.

2. Какой ток называется постоянным?

Величина и направление которого остаются неизменными

3. Какой ток называется переменным?

Величина и значение которого не остаются неизменными.

4. Что такое мгновенное значение тока?

Значение переменного тока в данный момент времени

5.Какой ток называется периодическим?

Ток, мгновенные значения которого повторяются через равные промежутки времени

6. Каким символом на схемах указывают положительное направление тока?

7. Каким символом на схемах указывают положительные направления ЭДС и напряжения?

За + направление ЭДС условно принято направление движения заряда внутри источника энергии (от – к +)

Такая же стрелка для + напряжений (напр. Уменьшения потенциалов)

8. Что такое цикл периодического тока?

Полный круг изменения переодического тока

9. Что называется периодом?

Время одного цикла периодического тока.

10. Что такое частота периодического тока?

Число периодов в секунду. f

11. Какой ток называется синусоидальным?

Периодический ток, кот. изменяется по гармоническому закону

12. Что такое амплитудное значение синусоидального тока?

Наибольшее значение синусоидального тока.

13. Что такое электрическая цепь?

Сов-ть устройств предназначенных для получения, передачи и преобразования в другие виды эл. энергии.

14. Из каких элементов состоит электрическая цепь?

Источник и приёмник Эл энергии, связанные соединительными проводами.

15. По каким признакам классифицируются электрические цепи?

По роду тока протекающего в цепях (цепи постоянного и переменного тока) По способу соединения элементов цепей (разветвлённые и неразветвлённые) По числу источников энергии (с одним и несколькими источниками) По роду приёмников (резистивные, индуктивные ёмкостные(линейн. и Нелин.)

16. Какая цепь называется линейной?

Эл. цепь составленная из линейных элементов

17.Какая цепь называется нелинейной?

Содержащая хотя бы один нелинейный элемент

18. Какими величинами определяется режим работы цепи?

Напряжением; током, кот. в ней протекает;

Мощностью, которая в ней рассеивается.

19. Какой режим работы называется режимом согласованной нагрузки?

При котором его внутреннее сопротивление равно сопротивлению внешней цепи.

20. Какой режим работы называется номинальным?

Если напряжение, ток и мощность источника соответствуют тем значениям, на которые он рассчитан заводом-изг.

21. Какой режим работы называется режимом короткого замыкания?

Если сопротивление приёмника равно 0. Ток короткого замыкания ограничен только величиной внутр. Сопротивления и во много раз превышает номинальный.

22. Какой режим работы называется режимом холостого хода источника?

напряжение на зажимах источника ЭДС может быть измерена при отключённом от источника приёмнике.

23. Какой режим работы называется рабочим?

Если ток, напряжение и мощность указанная заводом-изг. Не выдерживается, но отклонение от этих величен находится в допустимых пределах.

24. Как сформулировать и записать обобщенный закон Ома для цепи постоянного тока?

Ток в цепи пропорционален ЭДС источника и обратно пропорционален полному сопротивлению цепи.

25. Что называют параметрами элементов электрической цепи?

Коэф-ты, которые в уравнениях связывают токи и напряжения, характеризуют способность поглощать Эл. энергию и преобразовывать её в др. вид, создавая свои магнитные и Эл. поля.

26. Какое свойство элемента цепи характеризует параметр сопротивления?

Св-во необратимо преобразовывать электрическую энергию в др. виды

27. Какое свойство элемента цепи характеризует параметр индуктивности?

Хар-ет св-во элемента создавать своё собственное магнитное поле и накапливать в нём энергию при протекании через элемент тока.

28. Какое свойство элемента цепи характеризует параметр емкости?

Хар-ет св-во элемента создавать своё собственное Эл. поле и накапливать в нём энергию при протекании через элементтока

29. Как зависит мощность, рассеиваемая в элементе цепи, от сопротивления?

Параметр сопротивления явл. коэф-ом пропорциональности между мощностью преобразования Эл. энергии и квадратом протекающего через элемент тока.

30. Как зависит энергия, запасаемая в магнитном поле элемента цепи, от индуктивности?

31. Как зависит энергия, запасаемая в электрическом поле элемента цепи, от емкости?

32. Какой элемент цепи называется идеальным?

Если он обладает только одним параметром.

33. Какой элемент цепи называется реальным?

Элементы, обладающие больше, чем одним параметром.

34. Какими параметрами характеризуется источник энергии?

ЭДС; Внутр. Сопротивление

35. Какое свойство источника характеризует ЭДС?

хар-т св-во источника поддерживать разность потенциалов на зажимах цепи

36. Какое свойство источника характеризует его внутреннее сопротивление?

хар-ет св-во источника преобразовывать электр. Энергию, которую сам выработал в тепло

38. Что такое схема замещения электрической цепи?

Расчётная модель реального электротехнического устройства, графическое изображение реальной цепи с пом-ю идеальных элементов, пар-ы которых явл. Параметрами замещённых элементов

39. Какие элементы реальной цепи не включаются в схему замещения?

Указываются только те элементы, которые оказ. влияние на работу цепи в установившемся режиме.

40. Что такое узел, ветвь и контур электрической цепи?

Ветвь – участок, элементы которого соединены последовательно.

Узел – точка, в которой сходится не менее трёх ветвей

Контур – любой замкнутый путь для эл. тока.

41. Какой контур цепи называется независимым?

Контур, имеющий хотя бы один элемент, не принадлежащий другим контурам.

42. Какие ветви цепи называются параллельными?

Если они подключены к одной паре узлов.

43. Какие элементы цепи называются последовательными?

Если в цепи протекает один и тот же ток.

44. Как зависят напряжения на элементах последовательной цепи от их сопротивлений?

45. Как зависят токи, протекающие в элементах параллельной цепи, от их сопротивлений?

46. Как сформулировать и записать математически первый закон Кирхгофа?

Алгебраическая сумма токов в узле равна 0.

47. Как сформулировать и записать математически второй закон Кирхгофа?

Алгебраическая сумма электродвижущих сил, действующих в контуре, равна алгебраической сумме падений напряжений в том же контуре.

48. Какие токи и ЭДС берутся со знаком «-» при составлении уравнений на основании второго закона Кирхгофа?

Которые не совпадают с направлением обхода.

49. Сколько уравнений необходимо составить при расчете цепей методом непосредственного применения законов Кирхгофа?

Общее число уравнений равно числу неизвестных токов в цепи.

50. Сколько уравнений необходимо составить при расчете цепей методом контурных токов?

Общее число уравнений равно числу независимых контуров.

51. На каком принципе основан расчет цепей по методу наложения?

На принципе суперпозиций, применимо для линейных физических систем.

52. Сформулируйте принцип суперпозиции применительно к электрическим цепям.

Ток в любой ветви сложной цепи, содержащей неск. ЭДС равен алгебраической сумме токов от действия каждой из ЭДС отдельно.

53. Как составить уравнение баланса мощностей?

Согласно закону сохранения энергии.

54. В каком случае, при составлении баланса мощностей, произведение EI берется со знаком «+»?

Если направление ЭДС источника и напр. Тока в итой ветви цепи совпадают

57. Какой приемник называется активным?

Если при его работе в нём самом возникает ЭДС.

58. Запишите основное уравнение активного приемника?

№2 Линейные однофазные цепи синусоидального тока

1. По какой причине сопротивление элемента цепи постоянному току меньше активного сопротивления?

Из-за возникновения поверхностного эффекта и из-за возникновения вихревых токов в сердечниках (ток-синусоидальный)происх. увеличение затрат мощности и увеличение сопротивления по сравнению с устройствами пост. Тока.

2. Запишите уравнение для мгновенного значения синусоидального тока?

3. Что такое фаза синусоидального тока?

Угол, который вектор образует с горизонтальной осью (фазовый угол)

4. Что такое начальная фаза синусоидального тока?

Значение аргумента синусоидальной функции в момент начала отсчёта времени т. е. при t=0

5. Что такое угловая частота синусоидального тока?

Скорость изменения синусоидальной функции

6. Что называется действующим значением синусоидального тока?

Численно равно такому значению постоянного тока, которое за время одного периода переменного тока выделяет в элементе цепи такое же кол-во тепла, что ти синусоидальный ток.

7. Как связанны амплитудное и действующее значение синусоидального тока?

8. Запишите закон Ома для действующих значений тока и напряжения идеальной резистивной цепи синусоидального тока.

9. Чему равна разность фаз между током и напряжением идеальной резистивной цепи синусоидального тока?

0. ток в резестивной цепи так же как и напряжение меняется по закону синуса и в любй момент времени совпадает с напряжением по фазе.

10. Какой физический процесс протекает в цепи, содержащей только активное сопротивление?

Наличие поверхностного или скин-эффекта, зависящего от частоты (с увелич. частоты сопротивление растёт)

11. Как рассчитать среднюю за период мощность цепи, содержащей только активное сопротивление?

12. В чем причина, определяющая различие электромагнитных процессов цепей постоянного и переменного токов?

В том, что в одних полях расстояния

13. Что такое ЭДС самоиндукции?

Явление возникновения ЭДС индукции в проводящем контуре при любом изменении в нем силы тока

14. Какой физический процесс протекает в цепи синусоидального тока, содержащей только индуктивность?

Возникает ЭДС индукции; ЭДС тем больше, чем больше индуктивность и скорость изменения тока, т.е. частота, а следовательно

15. Запишите формулу для вычисления индуктивного сопротивления?

16. Как изменится индуктивное сопротивление при увеличении частоты?

при увеличении частоты, индуктивное сопротивление растёт

17. Запишите закон Ома для действующих значений тока и напряжения идеальной индуктивной цепи синусоидального тока?

18. Что такое комплекс индуктивного сопротивления?

Это отношение комплекса напряжения к комплексу тока. Он должен быть только положительным числом

19. Запишите закон Ома в комплексной форме для идеальной индуктивной цепи синусоидального тока.

20. Чему равна разность фаз между напряжением и током идеальной индуктивной цепи синусоидального тока?

Ток в идеальной индуктвной цепи, так же как и напряжение изменяется по закону синуса и отстаёт по фазе от напряжения на одну четверть периода, т.о. ток по фазе отстаёт от напряжения на 90

21. С какой частотой, по отношению к току, изменяется мгновенная мощность в идеальной индуктивной цепи синусоидального тока?

Изменяется с удвоенной частотой по отношению к току и напряжению

22. Как изменится ток в идеальной индуктивной цепи синусоидального тока при увеличении частоты?

При увеличении частоты ток в идеальной индуктивной цепи уменьшается

23. Чему равна средняя за период мощность идеальной индуктивной цепи синусоидального тока?

Средняя мощность за период равна нулю

24. Запишите формулу для расчета энергии, запасаемой в магнитом поле индуктивности, находящейся под синусоидальным напряжением.

25. Каким сопротивлением обладает элемент цепи, если ток, протекающий по нему и напряжение на нем совпадают по фазе?

Активное сопротивление

26. Каким сопротивлением обладает элемент цепи, если ток, протекающий по нему, отстает по фазе от напряжения на 90о?

Индуктивное сопротивление

27. Каким сопротивлением обладает элемент цепи, если ток, протекающий по нему, опережает напряжение по фазе на 90о?

Ёмкостное сопротивление

28. От каких факторов зависит проявление поверхностного эффекта?

Зависит от частоты (с увеличением частоты сопротивление растёт)

29. Какой физический процесс протекает в цепи синусоидального тока, содержащей только емкость?

Ток зарядки и разрядки не может быть неограниченно большим

30. Как вычислить емкостное сопротивление, если емкость и частота известны?

31. Как изменится емкостное сопротивление при увеличении частоты?

Уменьшится

32. Запишите закон Ома для действующих значений тока и напряжения идеальной емкостной цепи синусоидального тока.

33. Что такое комплекс емкостного сопротивления?

Величина обратная

34. Запишите закон Ома в комплексной форме для идеальной емкостной цепи синусоидального тока.

35. Чему равна разность фаз между током и напряжением идеальной емкостной цепи синусоидального тока?

36. С какой частотой, по отношению к току, изменяется мгновенная мощность в идеальной емкостной цепи синусоидального тока?

С удвоенной частотой по отношению к току и напряжению

37. Как изменится ток в идеальной емкостной цепи синусоидального тока при уменьшении частоты?

уменьшится

38. Чему равна средняя за период мощность идеальной емкостной цепи синусоидального тока?

0

39. Запишите формулу для расчета энергии, запасаемой в электрическом поле емкости, находящейся под синусоидальном напряжением.

40. Сформулируйте первый закон Кирхгофа для мгновенных значений синусоидальных токов.

алгебраическая сумма мгновенных значений токов в узле равна нулю

41. Сформулируйте второй закон Кирхгофа для мгновенных значений токов и напряжений.

Алгебраическая сумма напряжений на резистивных, индуктивных, ёмкостных элементах контура в данный момент времени равна алгебраической сумме ЭДС в том же контуре, в тот же момент времени.

42. Сформулируйте первый закон Кирхгофа для действующих значений токов.

Геометрическая сумма токов сходящихся в узле равна нулю

43. Сформулируйте второй закон Кирхгофа для действующих значений токов и напряжений.

Геометрическая сумма ЭДС, действующих в контуре, равна геометрической сумме падений напряжений в том же контуре

45. Как рассчитать действующее значение напряжения на зажимах последовательной цепи R-L-C, если напряжения на участках цепи известны?

47. Как рассчитать коэффициент мощности, если падения напряжений на участках последовательной цепи известны?

48. Как рассчитать падение напряжения на участке R последовательной цепи R-L-C, если напряжение на ее зажимах и угол сдвига фаз между током и напряжением известны?

49. Как рассчитать реактивное напряжение последовательной цепи R-L-C, если напряжение на ее зажимах и угол сдвига фаз между током и напряжением известны?

51. Как определить угол сдвига фаз между током и напряжением последовательной цепи, если параметры элементов известны?

52. Как рассчитать активное сопротивление последовательной цепи R-L-C, если модуль комплекса полного сопротивления и угол сдвига фаз между током и напряжением известны?

53. Как рассчитать реактивное сопротивление последовательной цепи R-L-C, если модуль комплекса полного сопротивления и угол сдвига фаз известны?

54. Что такое комплекс полного сопротивления?

55. Что такое модуль комплекса полного сопротивления?

57. Что такое полная мощность?

Максимально возможная мощность при заданных напряжении и токе (при отсутствии сдвига фаз между напряжением и током)

58. Что такое активная мощность? Какой физический процесс, протекающий в цепи, она характеризует?

Характеризует способность элемента необратимо преобразовывать эл энергию в другие виды энергии.

59. Что такое реактивная мощность? Какой физический процесс, протекающий в цепи, она характеризует?

Характеризует процессы обмена энергией между источником и полями приёмников

60. Как связаны активная, реактивная и полная мощности?

Связаны квадратурой

61. Какой физический смысл Cosφ?

он показывает, какая доля мощности источника необратимо преобразуется в др виды, в частности используется на выполнение полезной работы

62. Запишите формулу для расчета активной мощности.

63. Запишите формулу для расчета реактивной мощности.

64. Какой режим работы цепи синусоидального тока называется резонансом?

Режим работы при котором цепь, несмотря на наличие в ней реактивных элементов, ведёт себя как идеальная активная.

65. Запишите условия резонанса в последовательной цепи R-L-C.

66. Как рассчитать резонансную частоту?

67. Какими путями можно ввести цепь в резонанс?

Изменяя частоту, а при неизменной частоте изменяя параметры: напряжение питания, изменение индуктивности или ёмкости.

68. Как связаны активная, реактивная и полная мощности при резонансе?

т.к. и

69. Почему резонанс напряжений может стать причиной аварии?

При наличии больших реактивных сопротивлений, кот. окажутся враз больше, чем на зажимах цепи, может произойти пробой изоляции реактивных элементов и вывести их из строя.

70. Запишите закон Ома для режима резонанса напряжений.

71. Как будет меняться ток в цепи по мере приближения к режиму резонанса напряжений?

Возрастать и уже в самом режиме резонанса он будет максимален

72. Запишите закон Ома для последовательной цепи R-L-C для действующих значений тока и напряжений.

73. Запишите закон Ома для последовательной цепи R-L-C в комплексной форме.

74. Что такое комплекс полной проводимости?

Отношение комплексного действующего значения тока к комплексному действующему значению напряжения

75. Как рассчитать модуль комплекса полной проводимости параллельной цепи R-L-C?

76. Запишите формулу для расчета полной проводимости.

77. Запишите формулу для расчета реактивной проводимости.

78. Запишите закон Ома для действующих значений токов и напряжения параллельной цепи R-L-C.

81. Как найти ток в неразветвленной части параллельной цепи R-L-C, если токи в ветвях известны?

82. Как найти активную составляющую тока, если ток в неразветвленной части цепи и угол сдвига фаз между током и напряжением известны?

83. Как найти реактивную составляющую тока, если ток в неразветвленной части цепи и угол сдвига фаз между током и напряжением известны?

84. Как найти угол сдвига фаз между током в неразветвленной части цепи и напряжением на зажимах параллельной цепи R-L-C, если токи в ветвях известны?

85. Как найти токи в ветвях параллельной цепи R-L-C, если проводимости ветвей и приложенное напряжение известны?

86. Запишите условие резонанса токов.

Реактивные проводимости равны;

87. Как изменяется ток в неразветвленной части параллельной цепи по мере приближения к режиму резонанса токов?

Уменьшается, в самом режиме ток минимален

88. Как изменится коэффициент мощности цепи R-L при увеличении сопротивления R?

увеличится

89. Как изменится коэффициент мощности цепи R-L при подключении к ней конденсатора с XC<XL?

увеличится

№3 Трёхфазные цепи

1. Что такое фаза трехфазной системы?

Каждая из цепей, которая входит в трёхфазную систему

2. Какое соединение фаз приемника называется звездой?

Такое соединение, при котором концы фаз соединяются в один узел, который называется нулевой точкой, а начала фаз соединяются с источником

3. Какое соединение фаз приемника называется треугольником?

Такое соединение, при котором конец предидущ фазы соединяется с началом последующей фазы

4. Какое напряжение называется фазным?

Напряжение между началом и концом одной фазы

5. Какое напряжение называется линейным?

Напряжения между началами двух фаз

6. Какой ток называется фазным?

Ток, протекающий в фазах приёмника

7. Какой приемник называется симметричным?

Если комплексы полных сопротивлений его фаз равны

8. Какой ток называется линейным?

Ток, который протекает в проводах, соединяющих начала фаз источника с началами фаз приёмника

9. Почему при включении звездой несимметричного приемника необходим нейтральный провод, а при включении симметричного — можно обойтись без него?

Потому, что в несимметричном приёмнике происходит перекос фаз, а нейтральный провод служит для поддержания симметричного фазного напряжения

10. Как рассчитать ток в нейтральном проводе, если фазные токи известны?

11. Как измерить фазное напряжение приемника?

Вольтметр между началом и концом фазы

12. Как связаны фазные и линейные токи при включении приемника звездой с нейтральным проводом?

13. Как измерить линейное напряжение приемника?

Вольтметр между началами двух фаз

14. Как связаны фазные и линейные напряжения приемника при включении его звездой с нейтральным проводом?

15. Как связаны фазные и линейные токи при включении симметричного приемника треугольником?

16. Как связаны фазные и линейные напряжения при включении приемника треугольником?

17. Как рассчитать активную мощность симметричного трехфазного приемника?

18. Как рассчитать активную мощность несимметричного трехфазного приемника?

19. Как рассчитать реактивную мощность симметричного трехфазного приемника?

20. Как рассчитать реактивную мощность несимметричного трехфазного приемника?

21. Как рассчитать полную мощность симметричного трехфазного приемника?

22. Как измерить активную мощность симметричного трехфазного приемника одним ваттметром?

Измерить напряжение в одной фазе и умн на 3

23. Как измерить активную мощность несимметричного трехфазного приемника?

Измерить напряжение в каждой из фаз и сложить их значения

24. Чему равна разность фаз фазных напряжений симметричной трехфазной цепи?

25. Запишите выражения комплексов фазных напряжений симметричной трехфазной цепи.

№4 Магнитные цепи

1. Что такое магнитная цепь?

Часть электротехнического устройства, содержащая ферромагнитные тела, предназначенная для создания магнитного поля

2. Какую функцию выполняют магнитные цепи в электротехнических устройствах?

Усиливают магнитное поле и придаёт ему нужную конфигурацию

3. Почему сердечники электромагнитных аппаратов выполняют из ферромагнитных материалов?

Усилить магнитное поле

4. Что показывает относительная магнитная проницаемость?

Показывает во сколько раз ферромагнитная среда способна усилить магнитное поле

5. Сформулируйте словесно закон полного тока.

Интеграл от напряжённости магнитного поля по любому замкнутому контуру равен алгебраической сумме токов, пронизывающих данный контур.

6. Какая магнитная цепь называется однородной?

Если

7. Что характеризует площадь петли гистерезиса ферромагнитного материала?

Его полощадь пропорциональна энергии которая затрачивается на один цикл перемагничивания единицы объёма ферромагнитного материала

8. Что такое коэрцитивная сила?

Значение напряжённости внешнего поля, при которой ферромаг. Полностью размагничивается

9. Какие ферромагнитные материалы называются магнитомягкими, какие магнитожесткими?

Магнитомягкие имеют узкую петлю гистерезиса, что обуславливает малые потери на перемагничивание, они легко перемагничиваются.

Магнитожёсткие имеют высокую остаточную индукцию и коэрцетивную силу. Петля гистерезиса широкая.

10. Что изготавливают из магнитомягких материалов?

Листовая электротехническая сталь (магнитопроводы эл машин и аппаратов)

11. Что изготавливают из магнитожестких материалов?

Мостоянные магниты

№5 Трансформаторы

1. Что такое трансформатор?

Статический электромагнитный аппарат, предназначенный для преобразования переменного напряжения одной величины в переменное напряжение другой величины.

2. На каком законе физики основана работа трансформатора?

На законе электромагнитной индукции

3. Каково назначение трансформатора?

Предназначен для изменения напряжения

4. Из каких частей состоит трансформатор?

Замкнутый стальной стердечник, который выполнен из отдельных листов электротехнической стали, изолированных друг от друга слоем лака. НА сердечнике – как минимум две катушки, которые тщательно изолированы друг от друга и от сердечника.

5. Как связаны обмотки трансформатора между собой?

Вторичная является источником переменного U для первичной . Они не имеют электрической связи др. с др.

6. Каково назначение сердечника трансформатора?

Выполняет роль магнитопровода

7. Почему сердечник трансформатора изготавливается из отдельных изолированных листов электротехнической стали?

Для предотвращения появления вихревых токов

8. Почему сердечник трансформатора делается стальным?

Для лучшей проводимости тока

9. Что такое коэффициент трансформации трансформатора?

Отношение электродвижущих сил (отношение чисел витков первичной и вторичной обмоток)

10. Что такое номинальное напряжение обмотки?

Напряжение обмотки при холостом ходе трансформатора

11. Что такое номинальный ток обмотки?

Ток который связан с номинальной мощностью и номинальным напряжением таким соотношением

12. Что такое номинальная мощность трансформатора?

Мощность, которую трансформатор способен передать нагрузке не нагреваясь выше допустимой для него температуры

13. Какой режим работы трансформатора называется холостым ходом?

14. Запишите уравнение трансформаторной ЭДС.

Такой режим работы при котором первичная обмотка поставлена под ном. Напряжение, а вторичная разомкнута

15. Запишите уравнение электрического состояния первичной обмотки трансформатора.

16. Что такое приведенный первичный ток?

Составляющая первичного тока, компенсирующая размагничивающее действие вторичного тока.

17. Запишите уравнение первичного тока трансформатора.

18. Что такое намагничивающий ток трансформатора, как его измерить?

Ток холостого хода

19. Сформулируйте условия проведения опыта холостого хода трансформатора.

К первичной обмотке подключают номинальное напряжение, а ко вторичной – вольтметр, имеющий очень большое сопротивление, поэтому считают I2=0 a I1 намного меньше номинального

20. Что означает термин: «ток холостого хода 5%»?

Происходит падении напряжения в первичной обмотке, поэтому можно считать

21. Запишите уравнение электрического состояния вторичной обмотки трансформатора.

22. Сформулируйте условия проведения опыта короткого замыкания трансформатора.

Вторничная обмотка замыкается накоротко, а напряжение на первичной обмотке устанавливают такой величины, чтобы ток в ней был равен номинальному.

23. Как изменится магнитный поток в сердечнике трансформатора при увеличении тока вторичной обмотки?

увеличится

24. Что такое коэффициент загрузки трансформатора?

Бэтта – отношение тока, протекающего в обмотке трансформатора к номинальному току той же обмотке.

25. Как изменятся потери в сердечнике трансформатора при увеличении тока вторичной обмотки?

Не изменится

26. При каком условии КПД трансформатора достигает максимального значения?

Когда переменные потери в обмотках станут равными постоянным потерям в сердечнике

27. Как изменятся потери в обмотках трансформатора при увеличении тока первичной обмотки?

увеличится

28. Запишите формулу для расчета коэффициента загрузки, соответствующего максимальному КПД?

29. Запишите формулу для расчета КПД трансформатора.

30. Запишите формулу для расчета годового КПД трансформатора.

31. Что называется коэффициентом трансформации трехфазного трансформатора?

Для групп отношение линейных напряжений на первичной и вторичной сторонах равно коэффициенту трансформации. т.о. при соединении

При соединении

№9 Основы промышленной электроники

1. Что называется p-n переходом?

Граница между областями с различной проводимостью в одном кристалле полупроводника

2. Какое включение p-n перехода называется прямым?

Если источник подключён + к Р

3. Какое включение p-n перехода называется обратным?

Если источник подключён + к n

4. Как изменится сопротивление запирающего слоя при увеличении обратного напряжения?

Запирающий слой увеличивается, возрастает его сопротивление, ток протекать не будет.

5. Как изменится сопротивление запирающего слоя при увеличении прямого напряжения?

Толщина запирающего слоя и его сопротивление уменьшается

6. Каким свойством обладает p-n переход?

Свойство пропускать ток только в одном направлении (если + к Р)

7. Какой прибор называется диодом?

Полупроводниковый прибор с одним электрическим переходом и 2 внешними выводами

8. Начертите условное обозначение диода.

10. Что называют электрическим пробоем p-n перехода?

Резкое возрастание обратного тока даже при незначит увеличении обр. напряжения, свер определённого значения, при котором п-н переход не разрушается и сохраняет работоспособность.

11. Каково назначение выпрямителя?

Преобразования переменного напряжения в постоянное

12. Из каких основных частей состоит выпрямитель?

Силовой трансформатор, система вентелей (диодов), сгалживающий фильтр, стабилизатор

14. В каком соотношении находятся среднее выпрямленное напряжение однофазного однополупериодного выпрямителя и действующее значение напряжения на его входе?

15. В каком соотношении находятся обратное напряжение на диоде и среднее выпрямленное напряжение однофазного однополупериодного выпрямителя?

16. Как изменится величина выпрямленного напряжения при подключении емкостного фильтра?

Увеличивается

19. Какую роль играет фильтр в выпрямителе?

Для сглаживания пульсаций выпрямленного напряжения

20. Как изменится частота пульсаций выпрямленного напряжения при подключении емкостного фильтра?

Остаётся неизменной

21. Как изменится амплитуда пульсаций выпрямленного напряжения при подключении емкостного фильтра?

Увеличится

23. В каком соотношении находятся среднее выпрямленное напряжение однофазного мостового выпрямителя и действующее значение напряжения на его входе?

24. В каком соотношении находятся обратное напряжение на диоде и среднее выпрямленное напряжение однофазного мостового выпрямителя?

25. Что такое биполярный транзистор?

Полупроводниковый прибо, имеющий два п-н перехода и 3 внешних вывода

31. Как осуществляется управление выходным током в биполярном транзисторе?

Путём изменения входного тока

32. Как осуществляется управление выходным током в униполярном транзисторе?

Путём изменения ширины канала

33. Каков физический смысл параметров транзистора: h21, h22, h31, h32?

входное сопротивление (зависимость входного падения напр. От вх. Тока при пост. Вх. Напряж.)

коэф. Обратной связи (зависимость входного напряжения от выходного напряжения при пост. Входном токе.)

коэф. Передачи тока (зависимость выходного тока от входного при пост. Вых. напряжении)

выходная проводимость транзистора (определяет зависимость выходного тока от вых. Напряжения при пост вх токе)

34. Какой режим работы транзистора называется динамическим?

В котором к эмиттерному переходу приложено напряжение в прямом направлении (+ к эмиттеру)

36. Какую роль играют разделительные конденсаторы в усилительном каскаде?

1- защищает источник сигнала от попадания на него большого постоянного напряжения с резистора; 2 – разделяет составляющие коллекторного тока.

37. Что такое коэффициент усиления усилителя?

Отношение напряжения или тока (мощности) на выходе усилителя, к напряжению или току на его входе

38. Что такое полоса пропускания усилителя?

Диапазон частот

40. Объясните причины завалов амплитудно-частотной характеристики усилителя низкой частоты на высоких и низких частотах?

Ов низких част – наличие разделительных конденсаторов, кот по отношению к усилительному сигналу включён последовательно, а в высоких – обусловлен паразитной ёмкостью к-б перехода.

42. Почему амплитудная характеристика усилителя низкой частоты не выходит из начала координат?

Тк если на вход усилителя мы ничего не подщаём, то на выходе мы всёравно имеем какое-то напряжение

43. Что такое нелинейные искажения?

При больших входных сигналах () пропорциональность между входным и выходным напряжениями нарушается из-за нелинейности вольт-амперных характеристик транзистора. Это приводит к искажению формы выходного сигнала.

45. Что такое фазовые искажения усилителя и в чем их причины?

Несоответствие входных и выходных фаз. Причина – нелинейность характеристики транзистора

7 Асинхронный двигатель

1. Каково назначение статора асинхронного двигателя?

Служит для создания вращающего магнитного поля

2. Запишите формулу для расчета частоты вращения магнитного поля статора.

3. Почему ротор асинхронного двигателя набирается из отдельных изолированных листов стали?

Для того, чтобы не возникало вихревых токов

4. В чем отличие короткозамкнутого ротора от фазного?

К.з.- из Cu или Al стержни; Фазный- из Cu провода сведенного в звезду

5. Что такое скольжение асинхронного двигателя?

Отношение разности частот вращения поля статора и вращения ротора к частоте вращения поля статора.

6. При каком условии работа асинхронного двигателя устойчива?

Sн=0,02-0,08

7. Как рассчитать частоту тока в роторе?

8. Что понимают под номинальной мощностью двигателя?

Max. P , кот. потребляет двигатель

9. Как рассчитать номинальную мощность двигателя?

10. Как конструктивно выполнена обмотка ротора короткозамкнутого асинхронного двигателя?

В форме беличьего колеса

11. Чему равна предельная частота вращения асинхронного двигателя при частоте 400 Гц?

24000

12. Как рассчитать номинальный момент двигателя?

13. Что такое критическое скольжение?

14. Чему равна предельная частота вращения магнитного поля статора при частоте 50 Гц?

3000об/мин

16. Что означает термин: «рабочий участок механической характеристики жесткий»?

Т.е. в широком диапазоне нагрузок U меняется мало

17. С какой целью при пуске мощных асинхронных двигателей понижают напряжение?

Чтобы понизить ток

18. Что такое кратность пускового тока?

19. Что такое кратность пускового момента?

20. Что такое перегрузочная способность асинхронного двигателя?

21. Как изменится частота вращения асинхронного двигателя при увеличении частоты тока?

22. Как изменится частота вращения асинхронного двигателя при увеличении числа полюсов?

Увеличится

уменьшится

23. Какими путями можно изменить частоту вращения асинхронного двигателя?

Изменяя частоту тока в сети или число пар полюсов обмотки статора

24. Как изменить направление вращения асинхронного двигателя?

Поменяв местами 2 провода, соединяющие клеммы 3-фазной сети

25. Как изменится режим работы двигателя при обрыве одной из фаз?

Скорость вращения не изменится, а ток возрастет

Двигатели постоянного тока

1. Каково назначение статора двигателя постоянного тока?

Создание постоянного неподвижного магнитного поля

2. Почему якорь двигателя постоянного тока выполнен из отдельных изолированных листов стали?

Для уменьшения потерь на вихревые токи

3. Какую роль в двигателе постоянного тока играет коллектор?

Поддерживает направление вращающего момента

4. На каких законах физики основан принцип действия двигателя?

Закон электромагнитной индукции и закон ампера

5. Запишите основное уравнение двигателя постоянного тока.

6. Запишите уравнение ЭДС, возникающей в двигателе постоянного тока.

7. Запишите уравнение вращающего момента двигателя постоянного тока.

8. Запишите скоростное уравнение двигателя постоянного тока параллельного возбуждения.

9. С какой целью в цепь якоря двигателя постоянного тока включают пусковой реостат?

Для регулирования пускового якорного тока

10. Что такое кратность пускового тока?

Отношение пускового тока к номинальному.

11. Как изменится частота вращения двигателя постоянного тока при увеличении тока возбуждения?

увеличится

12. Как изменится частота вращения двигателя постоянного тока при увеличении магнитного потока?

уменьшится

13. Как изменится частота вращения двигателя постоянного тока при увеличении напряжения сети?

Увеличивается

14. Как изменить направление вращения двигателя постоянного тока?

Изменить полярность источника питания якорной цепи (цепи возбуждения)

15. Запишите уравнение механической характеристики двигателя постоянного тока параллельного возбуждения.

16. Запишите уравнение механической характеристики двигателя постоянного тока последовательного возбуждения.

17. Почему разрыв цепи возбуждения во время работы для двигателя постоянного тока параллельного возбуждения опасен?

Очень большой пусковой момент

18. Как изменится частота вращения двигателя постоянного тока последовательного возбуждения при уменьшении нагрузки на валу?

увеличивается

19. Почему работа двигателя постоянного тока последовательного возбуждения на холостом ходу недопустима?

Потому что на холл ходу скорость вращения вала может вырасти настолько, что центробежные силы вырвут якорные обмотки из пазов и двигатель выйдет из строя.

20. Как изменится частота вращения двигателя параллельного возбуждения при увеличении нагрузки на валу?

Остаётся неизменной

Раздел 8. Основы электрических измерений

1. Что такое измерение?

Познавательный процесс сравнения измеряемой величины с некоторым её значением принятым за единицу.

2. Что такое мера?

Вещественное воспроизведение единицы измеряемой физич величины с определяемой наперёд заданной точностью.

3. Что такое измерительный прибор?

Устройство, служащее для сравнения измеряемой величины с мерой

4. Что такое погрешность измерения?

Отклонение результатов измерения от действительного значения.

5. Что такое абсолютная погрешность измерения?

Погрешность, выраженная ыв единицах измеряемой величины Х-измеренное значение, Ха-действительное значение измеряемой величины.

6. Что такое относительная погрешность измерения?

Погрешность,выраженная в процентах от действительного значения.

7. Что такое приведенная погрешность измерения?

Выраженная в процентах от нормированного значения шкалы прибора

8. Что такое поправка к показаниям прибора?

Компенсация погрешности по знаку

9. Дайте определения систематической случайной и субъективной погрешностей.

Сис – остаются в процессе измерения постоянными или изменяются по опр закону; Случ – врез причин кот отчётливо не проявляются в единичном измерении. Суб – погрешности вследствии ошибки экспериментатора

10. Какие измерения называются прямыми?

Результаты которых следуют непосредственно из опытов

11. Какие измерения называются косвенными?

Результаты которых следуют из зависимости исконных от измеренных прямым способом величин .

12. В чем сущность метода непосредственной оценки?

Величина определяется непосредственно по градуированной шкале отсчётного устройства.

13. В чем сущность метода сравнения?

Метод в котором измеряемая величина сравнивается с мерой.

14. Что такое класс точности прибора?

Приведённое значение основной максимальной погрешности на шкале

15. Чем обусловлена основная погрешность прибора?

Конструкцией прибора

16. Чем обусловлена дополнительная погрешность прибора?

Отклонением условий эксплуатации от нормированных

18. В каком соотношении должны находиться сопротивления амперметра и цепи, в которую он включается?

Сопротивление амперметра не менее, чем на 3 порядка меньше

19. В каком соотношении должны находиться сопротивления вольтметра и участка цепи, параллельно которому он подключается?

Сопр. вольтметра больше соротивления участка цепи, параллельно которому он включается не менее, чем на три порядка

20. Как определить цену деления прибора?

Отношение диапазона измерения прибора к числу делений.

21. Как расширить пределы измерения магнитоэлектрических амперметров и вольтметров?

Для того, чтобы увеличить диапазон измерения, нужно последовательно подключить дополнительное сопротивление

22. Как расширяют пределы измерения электромагнитных амперметров и вольтметров?

Пределы электромаг. Вольтметров расширяют, используя измерительные трансформаторы

23. Как рассчитать сопротивление шунта?

I – предел измерения, до которого расшитряется диапазон

24. Как рассчитать величину добавочного сопротивления?

U – предел измерения, до которого расшитряется диапазон

25. Как рассчитать составляющие полной мощности при измерении методом трех приборов?

26. Какие резисторы называются малыми, средними, большими?

Малые до 1 Ом

Средние до 10 кОм

Большие больше 10 кОм

27. Начертите схему измерения сопротивлений малых резисторов методом двух приборов.

28. Начертите схему измерения сопротивлений больших резисторов методом двух приборов.

29. Начертите схему измерения сопротивлений в цепях переменного тока методом трех приборов.

31. Начертите схему измерения напряжения компенсационным методом.

32. Начертите схему измерения сопротивления мостовым методом.

Импеданс ведущего устройства LCR

Что такое импеданс?

Электрический импеданс (Z) — это общее противостояние, которое схема представляет для переменного тока. Импеданс изменяется в зависимости от компонентов в цепи и частоты применяемого переменного тока. Импеданс может включать сопротивление (R), индуктивное сопротивление (X L ) и емкостное сопротивление (X C ) . Это не просто алгебраическая сумма сопротивления, индуктивного реактивного сопротивления и емкостного сопротивления. Индуктивное реактивное сопротивление и емкостная реактивность 90 o вне фазы с сопротивлением, так что их максимальные значения происходят в разное время. Поэтому для вычисления импеданса необходимо использовать векторное сложение.

В цепи, подаваемой постоянным током, сопротивление представляет собой отношение приложенного напряжения (V) к результирующему току (I). Это Закон Ома.

 

Переменный ток регулярно меняет полярность.   Когда цепь переменного тока содержит только сопротивление, сопротивление цепи также определяется законом Ома.

Однако, когда емкость и / или индуктивность присутствуют в цепи переменного тока, они вызывают напряжение и ток не в фазе. Поэтому закон Ома должен быть изменен путем замены импеданса (Z) на сопротивление. Закон Ома становится: Z = V / I, где Z — комплексное число.

Z — комплексное число; т. е. имеет действительную компоненту (R) и мнимую компоненту ( jX ). Мнимая составляющая представляет собой любую точку на кривой переменного тока.

Сдвиг фазы

Сопротивление всегда находится в фазе с напряжением. Поэтому фазовый сдвиг всегда относительно линии сопротивления. Когда цепь имеет большее сопротивление по отношению к индуктивному сопротивлению, линия импеданса движется к линии сопротивления (ось X), и фазовый сдвиг уменьшается. Когда схема создает больше индуктивного сопротивления по отношению к сопротивлению, линия импеданса смещается в сторону индуктивной линии сопротивления (ось Y) и фазовый сдвиг увеличивается.

Импеданс в цепи с сопротивлением и индуктивным сопротивлением может быть рассчитан с использованием следующего уравнения. Если в цепи присутствовало емкостное сопротивление, его значение было бы добавлено к термину индуктивности перед возведением в квадрат.

Фазовый угол схемы можно рассчитать, используя приведенное ниже уравнение. Если в цепи присутствует емкостное реактивное сопротивление, его значение будет вычитаться из условия индуктивного сопротивления.

  Фазовый сдвиг может быть проведен на векторной диаграмме, показывающей последовательный импеданс, Z, его действительную часть Rs (последовательное сопротивление), его мнимую часть jXs (последовательное реактивное сопротивление) и фазовый угол θ.  

ω = 2πf

фигура 1 ,   Набор векторных диаграмм

Когда в цепи есть либо индуктивность, либо емкость, напряжение и ток не соответствуют фазе.

Индуктивность. Напряжение на индукторе является максимальным, когда скорость изменения тока больше. Для переменного (синусоидального) волнового типа это находится в точке, где фактический ток равен нулю. Напряжение, прикладываемое к индуктору, достигает своего максимального значения за четверть цикла до того, как ток будет протекать, а напряжение, как говорят, приводит ток на 90 o .

Емкость — ток, протекающий через конденсатор, прямо пропорционален величине самого конденсатора (заряд конденсатора с высоким значением заряда медленнее) и прямо пропорционален изменению напряжения конденсатора во времени. Ток, подаваемый на конденсатор, достигает максимального значения за четверть цикла до напряжения; ток приводит к напряжению на 90 o. по сравнению с конденсатором.

Серия против параллельных эквивалентов

Который должен быть измерен, серийный или параллельный параметры? Это зависит от цели измерения.    Для входящих проверок и производственных измерений на пассивные компоненты обычно значения серий указаны в стандартах EIA и MIL.   Эти стандарты также определяют тестовые частоты и другие условия испытаний.

Чтобы определить значение постоянного тока резистора с использованием измерений переменного тока, используйте последовательные измерения низкопотенциальных резисторов (скажем, под 1к) ; использовать параллельные измерения высокоценных.   В большинстве случаев это позволяет избежать ошибок из-за последовательной индуктивности и параллельной сосредоточенной емкости.   Кроме того, используйте низкую тестовую частоту.   Обратите внимание, что иногда измерение переменного тока может дать правильное значение постоянного тока лучше, чем измерение постоянного тока, потому что исключаются ошибки теплового напряжения и дрейфа, а чувствительность измерения выше.

Другими случаями, когда предпочтительны параллельные измерения, являются измерения очень низких значений емкости при проведении измерений на диэлектрических и магнитных материалах и, конечно же, при попытке параллельного определения отдельных значений двух компонентов.    Очень часто D конденсатора меньше 0,01, так что он не производит никакой разницы, которая измеряется, потому что разница между рядами и параллельными значениями меньше 0,01%.   Аналогично, резистор Q обычно меньше 0,01, так что можно измерять любое количество сопротивления.

Эквивалентная схема для этого импеданса поместила бы Rs и Xs последовательно, следовательно, индекс s ‘ .  

                         

Обратная Z является Впуск (Y), который также является комплексным число , имеющим действительную часть Gp (параллельно проводимости) и мнимая часть JBP (параллельно реактивным) с углом сдвига фаз ф.  

                

Полный список условий импеданса и уравнений см. На стр. 65 ,

Сопротивление, R, может быть задано с помощью одного действительного числа, а единица — Ом (Ω).   Сопротивление, G, устройства является обратным его сопротивлению: G = 1 / R.   Единицей проводимости является Siemen (ранее mho, «Ohm», записанный назад).  

Для переменного тока отношение напряжения к току является комплексным числом, поскольку напряжения и токи переменного тока имеют как фазу, так и величину.    Это комплексное число называется полным сопротивлением, Z и представляет собой сумму действительного числа R и мнимого, jX (где j = -1).   Таким образом, Z = R + jX .   Реальная часть — сопротивление переменного тока, а мнимая часть — реактивность.   У обоих есть единицы Ома.

Реакция происходит в двух типах: индуктивной и емкостной.   Реактивное сопротивление индуктивного элемента составляет L, где L — его индуктивность и    = 2πf (где f = частота).   Реактивное сопротивление емкостного элемента отрицательное, -1 / C, где C — его емкость.   Отрицательный знак возникает потому, что импеданс чистого конденсатора равен 1 / j C и 1 / j = -j.

Поскольку импеданс двух последовательно соединенных устройств представляет собой сумму их отдельных импедансов, рассмотрим импеданс как последовательную комбинацию идеального резистора и идеальный конденсатор или индуктор.   Это последовательная эквивалентная схема импеданса, содержащая эквивалентное последовательное сопротивление и эквивалентную последовательную емкость или индуктивность.   Используя индекс s для рядов, имеем:

Для сети, имеющей много компонентов, значения элемента эквивалентной схемы меняются с частотой.   Это также относится к значениям как индуктивных, так и емкостных элементов эквивалентной схемы одного действительного компонента (хотя изменения обычно очень малы).

Импеданс представлен на любой конкретной частоте эквивалентной схемой. Значения этих элементов или параметров зависят от того, какое представление используется, последовательно или параллельно, за исключением случаев, когда импеданс является чисто резистивным или чисто реактивным. В таких случаях необходим только один элемент, и ряды или параллельные значения одинаковы.

Впуск, Y, является обратным импедансом, как показано в уравнении 2:

Он также представляет собой комплексное число, имеющее реальную часть, проводимость переменного тока G и мнимую часть, восприимчивость B.   Поскольку добавление параллельных элементов добавляется, Y можно представить параллельной комбинацией идеальной проводимости и восприимчивости , где последняя является либо идеальной емкостью, либо идеальной индуктивностью.   Используя индекс p для параллельных элементов, мы имеем уравнение 3:

В общем случае Gp не равно 1 / Rs, а Bp не равно 1 / Xs (или -1 / Xs), как видно из расчета в уравнении 4.   

Таким образом, Gp = 1 / Rs, только если Xs = 0, что имеет место, только если импеданс является чистым сопротивлением; и Bp = -1 / Xs (обратите внимание на знак минуса), только если Rs = 0, т. е. импеданс является чистой емкостью или индуктивностью.

Две другие величины, D и Q, являются мерами «чистоты» компонента, т. Е. Насколько близко он идеален или содержит только сопротивление или реактивность.   D, коэффициент диссипации, представляет собой отношение действительной части импеданса или допуска к мнимой части. Q, коэффициент качества, является обратной величине этого отношения, как показано в уравнении 5.

Глубокое обсуждение «Истории измерений импеданса » Генри П. Холла — еще одна хорошо написанная статья по теме измерений импеданса.

ОСНОВНЫЕ ЭЛЕМЕНТЫ ЭЛЕКТРИЧЕСКОЙ ЦЕПИ СИНУСОИДАЛЬНОГО ТОКА —

Линейное активное сопротивление

При прохождении переменного тока через сопротивление происходит вытеснение тока к поверхности проводника, т. е. возникает поверхностный эффект. Эффективная площадь поперечного сечения уменьшается и его сопротивление увеличивается. Таким образом, величина сопротивления постоянному и переменному току не одинакова (для переменного тока она больше и увеличивается с увеличением частоты). Поэтому сопротивление при прохождении через него переменного тока называется не просто сопротивле- нием, а активным сопротивлением и обозначается буквой r.

Сопротивление при прохождении через него постоянного тока обозначается R и называется омическом сопротивлением.

Рис. 4.5.

Пусть через активное сопротивление r протекает синусоидальный ток U(t) = ImSinωt. Тогда напряжение U(t) будет тоже иметь синусоидальный характер U(l) = ImSinωt. В соответствии с законом Ома:

Обычно закон Ома записывают через действующие значения тока I и напряжения U

r = U / I

Начальные фазы тока и напряжения совпадают. Векторная диаграмма токов и напряжений имеет вид.

Рис.4.6.

Линейный индуктивный элемент

Индуктивный элемент преобразует энергию тока в энергию магнитного поля и может рассматриваться как аккумулятор энергии.

Линейный индуктивный элемент представляет преимущественно обмотку (катушку) в которой собственное потокосцепление пропорционально току.

Его параметром служит индуктивность L, а обозначение на рис. 4.6.

Рис. 4.7.

Пусть через катушку индуктивности L протекает синусоидальный ток i(t) = Im Sin ωt. В этом случае падение напряжения U(t) на катушке индуктивности будет определяться величиной ЭДС самоиндукции, т.е.

Закон Ома для амплитуд тока и напряжения имеет вид:

Um=ImωL; обозначая ωL= xL , получим

xL = ωL (4.11)

где, xL – индуктивное сопротивление

Начальная фаза напряжения ϕ = п /2, т.е. вектор напряжения сдвинут по фазе относительно вектора тока на угол 90′. В соответствии с этим векторная диаграмма будет иметь вид

UL

Рис. 4.8.

Линейный емкостной элемент

Емкостной элемент преобразует энергию напряжения электрического тока в энергию электрического поля и может рассматриваться как аккумулятор энергии. Линейный емкостной элемент представляет преимущественно конденсатор в котором значение заряда пропорционально напряжению.

Его параметром служит емкость С , а обозначение на рис. 4.7.

С

Рис. 4.9.

Пусть к емкости С подключено синусоидальное напряжение U(t) = UmSin ωt. В результате перемещения зарядов через конденсатор потечет переменный ток i(t) . Величина тока определяется по формуле:

Отсюда

Переходя от косинуса к синусу, получим

Вводя обозначение

получим:

Определим отношение Um / Im = l /ωC

Хс =l /ωC (4.12)

Хс – емкостное сопротивление

Вектор тока на конденсаторе опережает вектор напряжения на угол п /2.

Векторная диаграмма

п /2

Рис. 4.10.

Цепь с последовательным соединением элементов r, L, С.

Рис. 4.11.

Пусть i(t) = Im sinωt определим падения напряжения на всех элементах этой цепи. По второму закону Кирхгофа получим:

Изобразим векторную диаграмму токов и напряжений

Рис. 4.12.

UA=I r – активное напряжение; UP = UL-UC – реактивное напряжение

UL=I xL; UC=I xС (4.13)

(4.14)

Векторная диаграмма сопротивлений

Рис. 4.13.

ХP = ХLC – реактивное; Z – полное сопротивление

Z = √ r2+ ХP2 (4.15)

Векторная диаграмма мощностей; коэффициент мощности

Умножим каждую сторону треугольника напряжений на ток I. В результате получим векторная диаграмма мощностей (треугольник) мощностей.

Рис. 4.14.

Здесь: S – полная мощность, P-активная мощность, Q-реактивная мощность.

Полная мощность S – это мощность, вырабатываемая источником электро- энергии. Она состоит из двух составляющих: активной мощности Р и реактивной мощности Q. Причем величина полной мощности определяется по формуле:

(4.16)

Активная мощность Р источника расходуется на выполнение полезной работы (создание вращающего момента, нагрев и т.д.).

Реактивная мощность Q источника расходуется на перемагничивание катушек индуктивности L и перезарядку конденсаторов С. При этом происхо- дит процесс колебания энергии между источником и потребителями L и С.

Коэффициент мощности сosϕ определяет долю активной мощности в полной мощности, вырабатываемой источником сosϕ = Р / S.

Так как Р = S сosϕ , то из этого следует: чем больше величина сosϕ, тем больше мощности источника расходуется на полезную работу, т.е. тем больший кпд энергоустановки. При сosϕ= 1 вся мощность, вырабатываемая источником, расходуется на полезную работу. При малых значениях сosϕ для получения необходимой активной мощности необходимо иметь большое значение полной мощности, что связано с большими потерями электроэнергии, перерасходом металла и т.д. Таким образом коэффициент мощности – cosϕ определяет эффективность выполнения полезной работы.

Резонанс напряжений (при последовательным соединением r, L, С )

При xL= xC реактивное сопротивление равно нулю xP=0 и наступает резонанс напряжений UL= UC. При этом :

UL=UA , UP=0 , U=UA , Z = r , (4.17)

I=U/r = max; cosϕ=1 (4.18)

График изменения напряжения UL, UP, U при изменении частоты

ω0

Рис. 4.15.

Здесь ω0 – резонансная частота значения которой определяются по выра- жениям при резонансе ХL = ХC учитывая Хс =l /ωC; ХL = ωL получим:

l /ωC = ωL откуда ω0 l / C L (4.19)

Явление резонанса напряжений используется в радиотехнике для выделения частоты передачи и приема радиосигналов за счет I(ω0) = max;

а в электротехнике для повышения коэффициент мощности – cosϕ так как

cosϕ = 1, т.е. – максимальное значение.

Цепь с параллельным соединением элементов r, L, С.

Рис. 4.16.

В соответствии с первым законом Кирхгофа

I=IA+ IL+ IC

IA=U/r; IL=U/ωL; IC=UωC;

Построим векторную диаграмму токов и напряжения

Рис. 4.17.

(4.20)

(4.21)

В треугольнике токов разделим каждую сторону на напряжение, в результате получим векторную диаграмму (треугольник) проводимостей.

Рис. 4.18.

g- активная, b = b L– b C – реактивная, y – полная проводимости;

g = IA U; b = IP U = IL / U – IC / U; y = I / U ; y = √ g2 + b2 (4.22)

Векторная диаграмма мощностей

Умножим каждую сторону треугольника токов на напряжение U. В результате получим векторная диаграмма мощностей (треугольник) мощностей.

Рис. 4.19.

Здесь: S – полная мощность, P-активная мощность, Q-реактивная мощность.

Коэффициент мощности сosϕ определяет долю активной мощности в полной мощности, вырабатываемой источником сosϕ = Р / S.

Резонанс токов (при параллельным соединением r, L, C )

При bL= bC реактивная проводимость равна нулю b = 0 и наступает резонанс токов IC = IL При этом :

I = IA = U/r = min; cosϕ=1 (4.23)

График изменения токов при изменении частоты

ω0

Рис. 4.20.

Здесь ω0 – резонансная частота значения которой определяются по выра- жениям при резонансе bL = bC учитывая bс = ωC; bL = l /ωL получим:

ωC = l /ωL откуда ω0 l / C L (4.24)

Явление резонанса напряжений используется в радиотехнике для создания резонансных фильтров за счет I(ω0) = min; а в электротехнике для повышения коэффициент мощности – cosϕ так как cosϕ = 1, т.е. – максимальное значение.

Электрическое сопротивление и его виды

Основные понятия и определения электротехники

Любые устройства, служащие для получения, передачи или потребления электроэнергии, обладают сопротивлением.

Электрическое сопротивление это способность эле­мента электрической цепи противодействовать в той или иной степени прохождению по нему электрического тока. Сопротивление, в общем случае, зависит от материала эле­мента, его размеров, температуры, частоты тока и измеряется в омах (Ом). Различают активное (омическое), реактивное и полное сопротивления. Они обозначаются, соответственно, г, х, z. Используются также прописные буквы R, X, Z, чаще всего для обозначения элементов на электрических схемах:

 

 

Рис. 1.1. Электрическая схема цепи, содержащей два источника ЭДС с внутренними сопротивлениями R81 л R62, две активные и одну пассивную ветви,

соединенные в узлах а и Ь

Активное сопротивление элемента — это сопротивление постоянному току, Ом,

где р — удельное сопротивление материала, Ом-м,

 

а — температурный коэффициент сопротивления, °С»1;

t — интервал изменения температуры, °С;

/ — длина проводника, м;

5 — поперечное сечение проводника, м2.

Природу активного или омического сопротивления, связан­ного с нагревом материала, по которому протекает ток, объ­ясняют столкновением носителей заряда с узлами кристал­лической решетки этого материала.

Если электрическое сопротивление цепи или его элемента не зависит от величины проходящего тока, то такие цепи или элементы называют линейными. В противном случае говорят о нелинейных цепях.

Проводимость (активная) — величина обратная омичес­кому сопротивлению и измеряемая в сименсах (См):

 

В зависимости от величины удельной проводимости или

удельного сопротивления электротехнические материалы делят на проводники и диэлектрики или изоляторы (более подробные сведения в главах 3 и 4).

Индуктивное сопротивление — это сопротивление эле­мента, связанное с созданием вокруг него переменного или из­меняющегося магнитного поля. Оно зависит от конфигурации и размеров элемента, его магнитных свойств и частоты тока-

где xL — индуктивное сопротивление, Ом;

/ — частота тока, Гц;

со = Znf — угловая частота, рад/с;

L — индуктивность элемента цепи, (Гн).

Индуктивность можно определить как меру магнитной инерции элемента в отношении электромагнитного поля. По смыслу индуктивность в электротехнике можно уподобить массе в механике. Например, чем больше индуктивность элемента, тем медленнее и тем большую энергию магнитного поля он за­пасает.

Следует отметить, что индуктивным сопротивлением и, сле­довательно, индуктивностью обладают в разной мере все эле­менты электрической цепи переменного тока: обмотки электри­ческих машин, провода, шины, кабели и т. д. В цепях посто­янного тока индуктивное сопротивление проявляется лишь в переходных режимах.

Выражения для определения индуктивности элементов раз­личной конфигурации приведены в разделе 1.4.

Индуктивное сопротивление обозначается на электрических схемах:

где С —- электрическая емкость, Ф.

 

Емкостное сопротивление — это сопротивление элемента, связанное с созданием внутри и вокруг него электрического поля. Оно зависит от материала элемента, его размеров, конфигурации и частоты тока; измеряется в Омах (Ом):

Электрическую емкость можно определить как меру инертности элемента электрической цепи по отношению к электромагнитному полю. Электрическое поле между обклад­ками конденсатора создается вследствие разделения зарядов. Разделение зарядов происходит благодаря токам смещения, протекающим в диэлектрике между обкладки конденсатора под воздействием внешнего напряжения. Ток смещения следует понимать как процесс переориентации электрических диполей диэлектрика вдоль электромагнитного поля. Как видно, опреде­ление для тока, предложенное Фарадеем, наиболее привле­кательно для понимания сути токов смещения.

Таким образом, электромагнитная энергия аккумулируется в конденсаторе в виде энергии электрического поля, скон­центрированного в поляризованном диэлектрике между об­кладками конденсатора.

Если напряжение, приложенное к конденсатору, постоянно, то происходит его единичный заряд, после завершения которого ток через конденсатор, уменьшаясь, стремится к нулю. При перемен­ном напряжении происходит периодический перезаряд конденса­тора, поскольку токи смещения изменяют свой знак под воздейст­вием периодически изменяющего свой знак напряжения.

Практически все элементы электрической цепи переменного и постоянного тока в разной мере обладают емкостью. Для линий электропередач учет емкости поводов друг по отноше­нию к другу и по отношению к земле имеет принципиальное значение, поскольку влияет на режим электрических сетей. Например, обычные электрические кабели обладают емкост­ным сопротивлением порядка 10 Ом на 1 км.

На электрических схемах емкостные сопротивления обо­значаются:

 

 

 

 

 

 

Выражения для определения емкости элементов различной конфигурации приведены в разделе 1.4.

Реактивная проводимость, соответственно, делится на

 индуктивную, См,

и емкостную, См,



Мгновенная мощность цепи с ёмкости

ЦЕПЬ, СОДЕРЖАЩАЯ ЕМКОСТНЫЙ ЭЛЕМЕНТ С ЕМКОСТЬЮ С

В радиоэлектронных устройствах емкость является элементом колебательных контуров, фильтров, элементом связи между контурами и т. п. В силовых установках конденсаторы используют для улучшения коэффициента мощности, как элемент колебательного контура высокочастотных установок для закалки и плавки металлов. В любой электрической установке емкости образуются между проводами, проводами и землей и другими элементами токоведущих конструкций.

При большой протяженности проводов емкость может оказаться значительной, и при расчете цепей даже низкой, например промышленной, частоты ее необходимо учитывать. В высокочастотных цепях даже небольшие емкости оказывают существенное влияние на режим работы цепи и их необходимо учитывать.

Ток в цепи с емкостью (рис. 2.8, а) представляет собой движение зарядов к ее обкладкам:

Выразив в (2.10) заряд q через емкость С и напряжение на емкости иС, из выражения

Напряжение на емкости изменяется синусоидально:

Тогда ток в цепи

Взяв производную, получим мгновенное значение тока в цепи с емкостью:

Сравнивая выражения (2.11) и (2.12), можно сделать вывод, что ток в емкости опережает напряжение на емкости по фазе на 90°.

Векторная диаграмма цепи с емкостью приведена на рис. 2.8, б, а график мгновенных значений тока и напряжения — на рис. 2.8, в.

Рис. 2.8. Электрическая цепь, содержащая емкостный элемент с емкостью С (а), ее векторная диаграмма (б) и графики мгновенных значений u, i, p (в)

Напряжение и ток в цепи с емкостью, как следует из выражения (2.12), связаны соотношением

(2.13)

Разделив левую и правую части (2.13) на

, получим закон Ома для цепи с емкостью:(2.14)

где хС = 1/ωC — емкостное сопротивление, Ом.

Таким образом, напряжение на емкости в цепи переменного тока может быть выражено через произведение тока на емкостное сопротивление:

Мгновенное значение мощности р в цепи с емкостью равно произведению мгновенных значений напряжения и тока:

Из полученного выражения вытекает, что мгновенная мощность изменяется по закону синуса с частотой, в 2 раза большей частоты тока, и ее амплитудное значение

Среднее значение мощности за период (активная мощность), как видно из графика рис. 2.8, в, равно нулю:

Для пояснения энергетических процессов в цепях с емкостью воспользуемся графиками, изображенными на рис. 2.8, в. В первую четверть периода, в интервале времени между точками 1 и 2, напряжение на конденсаторе возрастает, происходит заряд конденсатора: электрическая энергия из сети поступает к конденсатору и накапливается в нем в виде энергии электрического поля. Накопленная энергия равна заштрихованной площади, ограниченной кривой р(t) (отмечена знаком « + »), и составляет

В следующую четверть периода, в интервале времени между точками 2 и 3, ток изменяет направление, а напряжение на конденсаторе убывает. Происходит разряд конденсатора: энергия электрического поля возвращается в сеть. Энергия, возвращенная в сеть, равна площади, ограниченной кривой р (t) (отмечена знаком « — »).

Из графиков рис. 2.8, в видно, что площади, определяющие запасенную и отданную энергии, равны. Следовательно, энергия, накопленная в электрическом поле емкости в первую четверть периода, полностью возвращается в сеть во вторую четверть периода.

В следующую четверть периода, в интервале времени между точками 3 и 4, изменяется полярность напряжения на обкладках конденсатора. Происходит заряд конденсатора: электрическая энергия из сети поступает к конденсатору и накапливается в нем в виде энергии электрического поля. В последнюю четверть периода, в интервале между точками 4 и 5, происходит разряд конденсатора: энергия электрического поля возвращается в сеть.

Таким образом, в цепи с емкостью, так же как и в цепи с индуктивностью, происходит непрерывный периодический процесс обмена энергией между сетью и конденсатором.

Не нашли то, что искали? Воспользуйтесь поиском:

Лучшие изречения: Для студентов недели бывают четные, нечетные и зачетные. 9634 —

| 7524 — или читать все.

188.64.169.166 © studopedia.ru Не является автором материалов, которые размещены. Но предоставляет возможность бесплатного использования. Есть нарушение авторского права? Напишите нам | Обратная связь.

Отключите adBlock!
и обновите страницу (F5)

очень нужно

Емкость и мощность электрической цепи

Энергия передается при помощи электрических цепей. В физике передача энергии обозначается $w$. Переход электромагнитной энергии в тепловую, в том числе процесс рассеивания энергии, характеризуется многочисленными преобразованиями и интенсивностью.

Интенсивность передачи или преобразования энергии принято называть мощностью. Она обозначается латинской буквой $p$.

Такие сложные процессы можно записать в виде математической формулы:

Иными словами, данным образом определяют, сколько энергии передается за определенную единицу времени. Тот же принцип действует при расчете рассеивания энергии за единицу времени.

Мгновенная мощность

Для мгновенного определения мощности в электрической цепи вводится формула в виде:

Мгновенная мощность обладает двумя видами элементов:

  • постоянной составляющей;
  • гармонической составляющей.

Мгновенная мощность имеет угловую частоту, которая превышает угловую частоту напряжения и тока в два раза. При отрицательных значениях мгновенной мощности энергия будет возвращаться к источнику питания. Это говорит о том, что направления напряжения и тока имеют противоположные значения в двухполюснике.

Попробуй обратиться за помощью к преподавателям

Подобное возвращение энергии к источнику происходит из-за того, что идет энергетический запас в электрических и магнитных полях на уровне емкостных и индуктивных элементов. Они входят в состав двухполюсника.

Активная мощность – среднее значение мгновенной мощности за определенный период времени.

Активная мощность при потреблении пассивным двухполюсником не имеет отрицательных значений. На входе пассивного двухполюсника будет фиксироваться $cos phi geq 0$. Ситуация, при которой $P=0$, возможна в теории, но только для двухполюсника без активных сопротивлений. В нем должны быть:

  • емкостные элементы;
  • идеальные индуктивные элементы.

Идеальная емкость

Конденсаторы являются идеальной емкостью в электрической цепи. В катушке индуктивности и конденсаторах активная мощность не потребляется, то есть $P=0$. В этот момент не происходит преобразования энергии в иные ее виды необратимого характера, а фиксируется лишь циркуляция имеющейся энергии. То есть электрическая энергия запасается в электрическом поле конденсатора или магнитном поле катушки. Процесс происходит примерно на протяжении 25 процентов времени периода. Затем энергия снова возвращается в сеть.

Задай вопрос специалистам и получи
ответ уже через 15 минут!

Конденсатор и катушку индуктивности из-за происходящих в них процессов иногда называют реактивными элементами. Их сопротивления принято называть реактивными. Исключение составляет резистор, так как он обладает активным сопротивлением.

Интенсивность обмена энергии характеризуется:

  • самым большим значением скорости поступления энергии в магнитное поле катушки;
  • наибольшей скоростью поступления энергии в электрическое поле конденсатора.

Эту интенсивность часто называют реактивной мощностью. Математическое выражение для реактивной мощности выглядит следующим образом:

При индуктивной нагрузке $phi geq 0$ реактивная мощность будет иметь положительные значения. При опережающем токе емкостной нагрузке – отрицательные.

Реактивная мощность для идеальной катушки индуктивности будет пропорциональна максимальному запасу энергии в катушке и частоте.

Коэффициент мощности – отношение полной мощности к активной мощности. Он равен косинусу угла сдвига между напряжением и током.

Помимо активной и полной мощности используют понятие комплексной мощности. Реактивная мощность характеризуется циркуляцией между потребителем и источником. Реактивный ток не совершает работу, что приводит к неоправданным потерям в силовом оборудовании. Это ведет к повышению уровня установленной мощности. Поэтому в настоящее время существует тенденция на увеличение мощности в электрических цепях.

Многие потребители в виде различных электродвигателей и иных приборов и устройств используют нагрузку активно-индуктивного характера. При условии подключения к подобной нагрузке конденсаторов общий ток потребителя приблизится к значениям фазы по напряжению. Это означает, что он увеличивается, но общая величина тока уменьшается при условии постоянной активной мощности. Этот факт приводит к потере общей величины тока в электрических цепях. Конденсаторы призваны повышать мощность.

Электрическая цепь с емкостью

Конденсаторы являются элементами электрической цепи, которые обладают значительной емкостью. Любые два проводника, расположенные рядом, имеют собственные емкости. При небольшой поверхности проводника емкость весьма небольшая, поэтому ее обычно не берут в расчет.

При рассмотрении электрической цепи выделяют основные ее элементы:

Конденсаторы являются идеальными диэлектриками, поэтому его активное сопротивление равно нулю. К электрической цепи с конденсатором подводят напряжение. Под его воздействием возникает ток. В это время на каждой стороне конденсатора начинает скапливаться заряд.

Емкость является главным параметром электрической цепи. Под емкостью между двумя телами называют отношение абсолютной величины заряда одного из тел к разности потенциалов, которая возникает между этими телами. Это действие обуславливает заряд в данных телах. Подобный процесс можно описать при помощи формулы в виде:

Емкость, как основной параметр электрической цепи, можно охарактеризовать в интегральной форме электрического поля участка цепи. Им является конденсатор. Подобная форма зависит от геометрических размеров, формы электродов, а также электрических свойств среды между электродами конденсатора.

Емкость измеряют в фарадах. Также применяются более мелкие единицы измерения (микрофарады, нанофарады и другие).

Так и не нашли ответ
на свой вопрос?

Просто напиши с чем тебе
нужна помощь

Мощность цепи.

Вырабатываемая мощность цепи переменного тока обладает свойством постоянно изменяться. Но при разделении периода такого тока и напряжения на маленькие временные промежутки величина тока и напряжения могут быть приняты как константы в данных временных промежутках. За каждый такой промежуток, который мы обозначим Δt, вырабатывается энергия, которая равна по величине средней величине тока, умноженной на среднюю величину напряжения, само собой, за данный промежуток времени:

Вообще говоря, ток и напряжение в цепи можно сместить по отношению друг к другу по фазе на произвольный угол, который мы назовем φ (рис. 1, а).

Рис. 1. Мощность переменного тока.

а — ток и напряжение смещены по фазе на угол φ;

б — ток и напряжение смещены по фазе на угол, равный 90°.

Будем считать началом временного отсчета момент, в который напряжение меняет свои значения с отрицательных на положительные. Тогда в момент, когда время равно нулю:

За маленький временной промежуток Δt в цепи выделится следующая энергия:

Тогда, если мы применим формулу из тригонометрии:

2 sinα • sinβ = cos(α — β) — cos(α + β),

то у нас получится:

За один период переменного тока будет выработана энергия, которая складывается из энергий, которые были выработаны за все маленькие промежутки времени, которые составляют период:

Так как величины, которые перемножаются в первом члене разности, константы, а во втором члене косинус в течение одной половины периода положителен, а вторую часть – отрицателен при тех же численных значениях, что в итоге дает ноль:

За один период средняя активная мощность цепи переменного тока равна:

В случае, когда ток и напряжение одинаковы по фазе, как если бы ток проходил через активное сопротивление:

Отсюда видно, что активная мощность цепи переменного тока равна мощности, которую вырабатывает постоянный ток, но только если его величина и величина напряжения будут меньше в √2 раз, чем амплитуда переменного. Такие значения переменного тока I и напряжения U называются действующими, или эффективными:

Так как в данной ситуации Um = Im•r, формула средней мощности цепи может быть выражена следующим образом:

Если взять конкретные амплитуды тока и напряжения, то вырабатываемая мощность цепи будет обратно зависеть от угла между их сдвигом фаз. Когда он равен 90° (рис. 1, б), а это аналогично цепям с реактивными элементами — идеальными конденсаторами и катушками индуктивности, работающими без потерь, средняя мощность цепи за один период будет равна нулю, потому что 25% периода будет предназначено для накопления энергии, а еще 25% — ее трате.

Условно отдаваемую и получаемую источником переменной э. д. с. мощность цепи называют реактивной, Рр. Если при этом будет происходить обмен энергией с реактивной нагрузкой (амплитуды тока и напряжения, умноженные на нагрузку на синус угла φ между ними и деленные пополам):

Примем во внимание, что напряжение на идеальной реактивной нагрузке Um = Im•X. Тогда:

Довольно распространен случай, когда на каком-то участке цепи существует переменное напряжение u = Um cosωt. Если через участок течет постоянный ток и токи различных частот, кратных ω, тогда:

Но какой энергетический эффект будет, если данные токи будут взаимодействовать с напряжением круговой частоты ω?

Ясно, что за один период средняя мощность взаимодействия постоянного тока с переменным напряжением равняется нулю. Первую половину периода она принимает положительные значения, так как источник расходует энергию, а вторую половину – отрицательные, так как источник получает ту же энергию обратно. Труднее понять, как происходит взаимодействие напряжения круговой частоты ω с токами кратных частот n•ω.

Чтобы вычислить среднюю за один период мощность напряжения T, необходимо, как и в прежних случаях, разделить период на маленькие временные участки Δt, на протяжении которых ток и напряжения можно принять за константы. Тогда мощность, достигнутая за такой промежуток, равна

Для вычисления средней мощности цепи за промежуток времени T, надо посчитать произведение всех pi на временные промежутки Δt, сложить их, а потом разделить на период T. В нашем случае будут суммироваться произведения вида

Ясно, что все такие суммы равняются нулю. На рис. 2 показаны напряжение и ток в ситуации, когда ток по частоте в два раза больше, чем напряжение (n = 2), и приведен график произведений их мгновенных значений. Если посмотреть на график произведений мгновенных значений внимательно, станет ясно, что мгновенная мощность цепи тоже изменяется во времени с определенным периодом и за время T два раза переходит от положительных к таким же, но отрицательным значениям. Так что средняя мощность цепи за временной интервал T будет равняться нулю. Ясно, что при всяком сочетании кратных частот будет наблюдаться та же картина.

Рис. 2. Мощность взаимодействия тока и напряжения кратных частот.

Основываясь на информации, которая была изложена в данном параграфе, можно сделать следующий вывод: если в цепи источника переменного напряжения протекают постоянный ток и переменные токи кратных частот, то энергетическое взаимодействие происходит только с током, частота которого равна частоте источника напряжения; источник постоянного напряжения дает эффект энергетического взаимодействия только с постоянной составляющей проходящего через него тока.

Цепь переменного тока с емкостью

Дата публикации: 31 марта 2015 .
Категория: Электротехника.

Если в цепь постоянного тока включить конденсатор (идеальный – без потерь), то в течение короткого времени после включения по цепи потечет зарядный ток. После того как конденсатор зарядится до напряжения, соответствующего напряжению источника, кратковременный ток в цепи прекратится. Следовательно, для постоянного тока конденсатор представляет собой разрыв цепи или бесконечно большое сопротивление.

Если же конденсатор включить в цепь переменного тока, то он будет заряжаться попеременно то в одном, то в другом направлении.

При этом в цепи будет проходить переменный ток. Рассмотрим это явление подробнее.

В момент включения напряжение на конденсаторе равно нулю. Если включить конденсатор к переменному напряжению сети, то в течение первой четверти периода, когда напряжение сети будет возрастать (рисунок 1), конденсатор будет заряжаться.

Рисунок 1. Графики и векторная диаграмма для цепи переменного тока, содержащей емкость

По мере накопления зарядов на обкладках конденсатора напряжение конденсатора увеличивается. Когда напряжение сети к концу первой четверти периода достигнет максимума, заряд конденсатора прекращается и ток в цепи становится равным нулю.

Ток в цепи конденсатора можно определить по формуле:

где q – количество электричества, протекающее по цепи.

Из электростатики известно:

где C – емкость конденсатора; u – напряжение сети; uC – напряжение на обкладках конденсатора.

Окончательно для тока имеем:

Из последнего выражения видно, что, когда

максимально (положения а, в, д), i также максимально. Когда (положения б, г на рисунке 1), то i также равно нулю.

Во вторую четверть периода напряжение сети будет уменьшаться, и конденсатор начнет разряжаться. Ток в цепи меняет свое направление на обратное. В следующую половину периода напряжение сети меняет свое направление и наступает перезаряд конденсатора и затем снова его разряд. Из рисунка 1 видно, что ток в цепи с емкостью в своих изменениях опережает по фазе на 90°

напряжение на обкладках конденсатора.

Сравнивая векторные диаграммы цепей с индуктивностью и емкостью, мы видим, что индуктивность и емкость на фазу тока влияют прямо противоположно.

Поскольку мы отметили выше, что скорость изменения тока пропорциональна угловой частоте ω, из формулы

получаем аналогично, что скорость изменения напряжения также пропорциональна угловой частоте ω и для действующего значения тока имеем

Обозначая

, где xC называется емкостным сопротивлением, или реактивным сопротивлением емкости. Итак мы получили формулу емкостного сопротивления при включении емкости в цепи переменного тока. Отсюда, на основании выражения закона Ома, мы можем получить ток для цепи переменного тока, содержащей емкость:

Напряжение на обкладках конденсатора

Та часть напряжения сети, которая имеется на конденсаторе, называется емкостным падением напряжения, или реактивной слагающей напряжения, и обозначается UC.

Емкостное сопротивление xC, так же как индуктивное сопротивление xL, зависит от частоты переменного тока.

Но если с увеличением частоты индуктивное сопротивление увеличивается, то емкостное сопротивление, наоборот, будет уменьшаться.

Пример 1. Определить емкостное реактивное сопротивление конденсатора емкостью 5 мкФ при разных частотах сетевого напряжения. Расчет емкостного сопротивления произведем при частоте 50 и 40 Гц:

при частоте 50 Гц:

при частоте 400 Гц:

Применим формулу средней или активной мощности для рассматриваемой цепи:

Так как в цепи с емкостью ток опережает напряжение на 90°, то

Поэтому активная мощность также равна нулю, то есть в такой цепи, как и в цепи с индуктивностью, расхода мощности нет.

На рисунке 2 показана кривая мгновенной мощности в цепи с емкостью. Из чертежа видно, что в первую четверть периода цепь с емкостью забирает из сети энергию, которая запасается в электрическом поле конденсатора.

Рисунок 2. Кривая мгновенной мощности в цепи с емкостью

Энергию, запасаемую конденсатором к моменту прохождения напряжения на нем через максимум, можно определить по формуле:

В следующую четверть периода конденсатор разряжается на сеть, отдавая ей ранее запасенную в нем энергию.

За вторую половину периода явление колебаний энергии повторяется. Таким образом, в цепи с емкостью происходит лишь обмен энергией между сетью и конденсатором без потерь.

Источник: Кузнецов М. И., «Основы электротехники» — 9-е издание, исправленное — Москва: Высшая школа, 1964 — 560 с.

Реактивное сопротивление, индуктивное и емкостное | Физика

Цели обучения

К концу этого раздела вы сможете:

  • Зависимость напряжения и тока от времени в простых индуктивных, емкостных и резистивных цепях.
  • Рассчитайте индуктивное и емкостное сопротивление.
  • Рассчитывайте ток и / или напряжение в простых индуктивных, емкостных и резистивных цепях.

Многие цепи также содержат конденсаторы и катушки индуктивности в дополнение к резисторам и источнику переменного напряжения.Мы видели, как конденсаторы и катушки индуктивности реагируют на постоянное напряжение при его включении и выключении. Теперь мы исследуем, как катушки индуктивности и конденсаторы реагируют на синусоидальное переменное напряжение.

Катушки индуктивности и индуктивное сопротивление

Предположим, индуктор подключен непосредственно к источнику переменного напряжения, как показано на рисунке 1. Разумно предположить, что сопротивление пренебрежимо мало, поскольку на практике мы можем сделать сопротивление индуктора настолько малым, что оно окажет незначительное влияние на схему.Также показан график зависимости напряжения и тока от времени.

Рис. 1. (a) Источник переменного напряжения, включенный последовательно с катушкой индуктивности, имеющей незначительное сопротивление. (б) График зависимости тока и напряжения на катушке индуктивности от времени.

График на Рисунке 1 (b) начинается с максимального напряжения. Обратите внимание, что ток начинается с нуля и повышается до своего пика после напряжения, которое им управляет, точно так же, как это было в случае, когда напряжение постоянного тока было включено в предыдущем разделе.Когда напряжение становится отрицательным в точке а, ток начинает уменьшаться; оно становится нулевым в точке b, где напряжение является самым отрицательным. Затем ток становится отрицательным, снова вслед за напряжением. Напряжение становится положительным в точке c и начинает делать ток менее отрицательным. В точке d ток проходит через ноль, когда напряжение достигает своего положительного пика, чтобы начать следующий цикл. Кратко это поведение можно описать следующим образом:

Напряжение переменного тока в индукторе

Когда на катушку индуктивности подается синусоидальное напряжение, оно опережает ток на одну четверть цикла или на фазовый угол 90º.

Ток отстает от напряжения, поскольку индукторы препятствуют изменению тока. Изменение тока индуцирует обратную ЭДС В = — L I / Δ t ). Это считается эффективным сопротивлением катушки индуктивности переменному току. Среднеквадратичный ток I через катушку индуктивности L определяется версией закона Ома:

[латекс] I = \ frac {V} {{X} _ {L}} \\ [/ latex],

, где В, — среднеквадратичное значение напряжения на катушке индуктивности, а X L определяется как

.

[латекс] {X} _ {L} = 2 \ pi {fL} \\ [/ латекс],

с f частота источника переменного напряжения в герцах (анализ схемы с использованием правила петли Кирхгофа и вычислений фактически дает это выражение). X L называется индуктивным реактивным сопротивлением , потому что катушка индуктивности препятствует прохождению тока. X L имеет единицы измерения Ом (1 Гн = 1 Ом с, так что частота, умноженная на индуктивность, имеет единицы (циклы / с) (Ом ⋅ с) = Ом)), что соответствует его роли в качестве эффективное сопротивление. Имеет смысл, что X L пропорционально L , поскольку чем больше индукция, тем больше его сопротивление изменению.Также разумно, что X L пропорционально частоте f , поскольку большая частота означает большее изменение тока. То есть Δ I / Δ t является большим для больших частот (большие f , маленькие Δ t ). Чем больше изменение, тем больше сопротивление катушки индуктивности.

Пример 1. Расчет индуктивного сопротивления, а затем тока

(a) Рассчитайте индуктивное сопротивление 3.Индуктор 00 мГн при подаче переменного напряжения 60,0 Гц и 10,0 кГц. (b) Каков среднеквадратичный ток на каждой частоте, если приложенное действующее напряжение составляет 120 В?

Стратегия

Индуктивное реактивное сопротивление находится непосредственно из выражения X L = 2πf L . Как только X L было найдено на каждой частоте, закон Ома, как указано в уравнении I = В / X L , может быть использован для определения тока на каждой частоте.

Решение для (а)

Ввод частоты и индуктивности в уравнение X L = 2πf L дает

X L = 2πf L = 6,28 (60,0 / с) (3,00 мГн) = 1,13 Ом при 60 Гц.

Аналогично, на 10 кГц,

X L = 2πf L = 6,28 (1,00 × 10 4 / с) (3,00 мГн) = 188 Ом при 10 кГц.

Решение для (b)

Среднеквадратичное значение тока теперь определяется с использованием версии закона Ома в уравнении I = В / X L , при условии, что приложенное действующее напряжение составляет 120 В.Для первой частоты это дает

[латекс] I = \ frac {V} {{X} _ {L}} = \ frac {120 \ text {V}} {1.13 \ text {} \ Omega} = 106 \ text {A at} 60 \ текст {Hz} \\ [/ latex].

Аналогично, на 10 кГц,

[латекс] I = \ frac {V} {{X} _ {L}} = \ frac {120 \ text {V}} {188 \ text {} \ Omega} = 0,637 \ text {A at} 10 \ текст {кГц} \\ [/ latex].

Обсуждение

Катушка индуктивности по-разному реагирует на двух разных частотах. На более высокой частоте его реактивное сопротивление велико, а ток невелик, что соответствует тому, как катушка индуктивности препятствует быстрому изменению.Таким образом, наиболее затруднены высокие частоты. Индукторы могут использоваться для фильтрации высоких частот; например, большую катушку индуктивности можно включить последовательно с системой воспроизведения звука или последовательно с вашим домашним компьютером, чтобы уменьшить высокочастотный звук, выводимый из ваших динамиков или высокочастотные всплески мощности на ваш компьютер.

Обратите внимание, что, хотя сопротивлением в рассматриваемой цепи можно пренебречь, переменный ток не очень велик, поскольку индуктивное реактивное сопротивление препятствует его протеканию.С переменным током нет времени, чтобы ток стал слишком большим.

Конденсаторы и емкостное сопротивление

Рассмотрим конденсатор, подключенный непосредственно к источнику переменного напряжения, как показано на рисунке 2. Сопротивление такой цепи можно сделать настолько малым, что оно окажет незначительное влияние по сравнению с конденсатором, поэтому мы можем предположить, что сопротивление незначительно. Напряжение на конденсаторе и ток показаны на рисунке как функции времени.

Рисунок 2.(а) Источник переменного напряжения, включенный последовательно с конденсатором С, имеющим незначительное сопротивление. (б) График зависимости тока и напряжения на конденсаторе от времени.

График на Рисунке 2 начинается с максимального напряжения на конденсаторе. В этот момент ток равен нулю, потому что конденсатор полностью заряжен и останавливает поток. Затем напряжение падает, а ток становится отрицательным по мере разряда конденсатора. В точке а конденсатор полностью разряжен (на нем Q = 0 ) и напряжение на нем равно нулю.Ток остается отрицательным между точками a и b, вызывая обратное напряжение на конденсаторе. Это завершается в точке b, где ток равен нулю, а напряжение имеет самое отрицательное значение. Ток становится положительным после точки b, нейтрализуя заряд конденсатора и доводя напряжение до нуля в точке c, что позволяет току достичь своего максимума. Между точками c и d ток падает до нуля, когда напряжение достигает своего пика, и процесс начинает повторяться. На протяжении всего цикла напряжение соответствует тому, что делает ток, на четверть цикла:

Напряжение переменного тока в конденсаторе

Когда на конденсатор подается синусоидальное напряжение, оно следует за током на одну четверть цикла или на фазовый угол 90º.

Конденсатор влияет на ток, имея возможность полностью его отключить, когда он полностью заряжен. Поскольку применяется переменное напряжение, возникает среднеквадратичный ток, но он ограничивается конденсатором. Это считается эффективным сопротивлением конденсатора переменному току, поэтому среднеквадратичный ток I в цепи, содержащей только конденсатор C , определяется другой версией закона Ома как

.

[латекс] I = \ frac {V} {{X} _ {C}} \\ [/ latex],

, где В, — среднеквадратичное значение напряжения, а X C определено (Как и в случае с X L , это выражение для X C является результатом анализа цепи используя правила и исчисление Кирхгофа) равным

[латекс] {X} _ {C} = \ frac {1} {2 \ pi fC} \\ [/ latex],

, где X C называется емкостным реактивным сопротивлением , потому что конденсатор препятствует прохождению тока. X C имеет единицы измерения Ом (проверка оставлена ​​в качестве упражнения для читателя). X C обратно пропорциональна емкости C ; Чем больше конденсатор, тем больший заряд он может накапливать и тем больше может протекать ток. Она также обратно пропорциональна частоте f ; чем выше частота, тем меньше времени остается для полной зарядки конденсатора и, следовательно, меньше препятствует току.

Пример 2. Расчет емкостного реактивного сопротивления, а затем тока

(a) Рассчитайте емкостное реактивное сопротивление конденсатора 5,00 мФ при приложении переменного напряжения 60,0 Гц и 10,0 кГц. (b) Каков среднеквадратичный ток, если приложенное действующее напряжение составляет 120 В?

Стратегия

Емкостное реактивное сопротивление находится непосредственно из выражения в [latex] {X} _ {C} = \ frac {1} {2 \ pi fC} \\ [/ latex]. После того, как X C было обнаружено на каждой частоте, закон Ома, сформулированный как I = В, / X C , можно использовать для определения тока на каждой частоте.

Решение для (а)

Ввод частоты и емкости в [латекс] {X} _ {C} = \ frac {1} {2 \ pi fC} \\ [/ latex] дает

[латекс] \ begin {array} {lll} {X} _ {C} & = & \ frac {1} {2 \ pi fC} \\ & = & \ frac {1} {6.28 \ left (60.0 / \ text {s} \ right) \ left (5.00 \ text {} \ mu \ text {F} \ right)} = 531 \ text {} \ Omega \ text {at} 60 \ text {Hz} \ end {массив }\\[/латекс].

Аналогично, на 10 кГц,

[латекс] \ begin {array} {lll} {X} _ {C} & = & \ frac {1} {2 \ pi fC} = \ frac {1} {6.{4} / \ text {s} \ right) \ left (5,00 \ mu \ text {F} \ right)} \\ & = & 3,18 \ text {} \ Omega \ text {at} 10 \ text {кГц} \ end {array} \\ [/ latex].

Решение для (b)

Среднеквадратичное значение тока теперь определяется с использованием версии закона Ома в I = В / X C , учитывая приложенное действующее напряжение 120 В. Для первой частоты это дает

[латекс] I = \ frac {V} {{X} _ {C}} = \ frac {120 \ text {V}} {531 \ text {} \ Omega} = 0,226 \ text {A at} 60 \ текст {Hz} \\ [/ latex].

Аналогично, на 10 кГц,

[латекс] I = \ frac {V} {{X} _ {C}} = \ frac {120 \ text {V}} {3.18 \ text {} \ Omega} = 3.37 \ text {A at} 10 \ текст {Hz} \\ [/ latex].

Обсуждение

Конденсатор очень по-разному реагирует на двух разных частотах, а индуктор реагирует прямо противоположным образом. На более высокой частоте его реактивное сопротивление мало, а ток велик. Конденсаторы одобряют изменения, тогда как индукторы противодействуют изменениям. Конденсаторы больше всего препятствуют низким частотам, так как низкая частота позволяет им успеть зарядиться и остановить ток.Конденсаторы можно использовать для фильтрации низких частот. Например, конденсатор, включенный последовательно с системой воспроизведения звука, избавляет ее от гула 60 Гц.

Хотя конденсатор в основном представляет собой разомкнутую цепь, в цепи с напряжением переменного тока, приложенным к конденсатору, присутствует среднеквадратичный ток. Это связано с тем, что напряжение постоянно меняет направление, заряжая и разряжая конденсатор. Если частота стремится к нулю (постоянный ток), X C стремится к бесконечности, и ток равен нулю, когда конденсатор заряжен.На очень высоких частотах реактивное сопротивление конденсатора стремится к нулю — он имеет незначительное реактивное сопротивление и не препятствует току (он действует как простой провод). Конденсаторы оказывают противоположное влияние на цепи переменного тока, чем индукторы .

Резисторы в цепи переменного тока

В качестве напоминания рассмотрим Рисунок 3, на котором показано напряжение переменного тока, приложенное к резистору, и график зависимости напряжения и тока от времени. Напряжение и ток равны точно в фазе в резисторе.Отсутствует частотная зависимость поведения простого сопротивления в цепи:

Рис. 3. (a) Источник переменного напряжения, включенный последовательно с резистором. (b) График зависимости тока и напряжения на резисторе от времени, показывающий, что они точно совпадают по фазе.

Напряжение переменного тока в резисторе

Когда на резистор подается синусоидальное напряжение, напряжение точно совпадает по фазе с током — они имеют фазовый угол 0 °.

Сводка раздела

  • Для катушек индуктивности в цепях переменного тока мы обнаруживаем, что когда на катушку подается синусоидальное напряжение, напряжение опережает ток на одну четверть цикла или на фазовый угол 90 °.
  • Противодействие катушки индуктивности изменению тока выражается как сопротивление переменному току.
  • Закон Ома для катушки индуктивности

    [латекс] I = \ frac {V} {{X} _ {L}} \\ [/ latex],

    , где В, — среднеквадратичное значение напряжения на катушке индуктивности.

  • X L определяется как индуктивное реактивное сопротивление, определяемое по формуле

    [латекс] {X} _ {L} = 2 \ pi fL \\ [/ латекс],

    с f частота источника переменного напряжения в герцах.

  • Индуктивное реактивное сопротивление X L выражается в единицах Ом и имеет наибольшее значение на высоких частотах.
  • Для конденсаторов мы обнаруживаем, что когда на конденсатор подается синусоидальное напряжение, напряжение следует за током на одну четверть цикла или на фазовый угол 90º.
  • Поскольку конденсатор может останавливать ток при полной зарядке, он ограничивает ток и предлагает другую форму сопротивления переменному току; Закон Ома для конденсатора

    [латекс] I = \ frac {V} {{X} _ {C}} \\ [/ latex],

    , где В, — среднеквадратичное значение напряжения на конденсаторе.

  • X C определяется как емкостное реактивное сопротивление, определяемое по формуле

    [латекс] {X} _ {C} = \ frac {1} {2 \ pi fC} \\ [/ latex].

  • X C имеет единицы измерения Ом и имеет наибольшее значение на низких частотах.

Концептуальные вопросы

1. Пресбиакузис — это возрастная потеря слуха, которая постепенно влияет на высокие частоты. Усилитель слухового аппарата предназначен для равномерного усиления всех частот. Чтобы отрегулировать его мощность на пресбиакузис, включите ли вы конденсатор последовательно или параллельно динамику слухового аппарата? Объяснять.

2. Будете ли вы использовать большую индуктивность или большую емкость последовательно с системой для фильтрации низких частот, таких как гул 100 Гц в звуковой системе? Объяснять.

3. Высокочастотный шум в сети переменного тока может повредить компьютеры. Использует ли съемный блок, предназначенный для предотвращения этого повреждения, большую индуктивность или большую емкость (последовательно с компьютером) для фильтрации таких высоких частот? Объяснять.

4. Зависит ли индуктивность от тока, частоты или и того, и другого? А как насчет индуктивного сопротивления?

5. Объясните, почему конденсатор на рисунке 4 (а) действует как фильтр низких частот между двумя цепями, тогда как конденсатор на рисунке 4 (b) действует как фильтр высоких частот.

Рисунок 4. Конденсаторы и катушки индуктивности. Конденсатор с высокой и низкой частотой.

6. Если конденсаторы на рис. 4 заменить катушками индуктивности, что будет действовать как фильтр низких частот, а какой — как фильтр высоких частот?

Задачи и упражнения

1. На какой частоте индуктор 30,0 мГн будет иметь реактивное сопротивление 100 Ом?

2. Какое значение индуктивности следует использовать, если требуется реактивное сопротивление 20,0 кОм при частоте 500 Гц?

3.Какую емкость следует использовать для получения реактивного сопротивления 2,00 МОм при 60,0 Гц?

4. На какой частоте конденсатор 80,0 мФ будет иметь реактивное сопротивление 0,250 Ом?

5. (a) Найдите ток через катушку индуктивности 0,500 H, подключенную к источнику переменного тока 60,0 Гц, 480 В. (б) Каким будет ток на частоте 100 кГц?

6. (a) Какой ток протекает, когда источник переменного тока 60,0 Гц, 480 В подключен к конденсатору 0,250 мкФ? (b) Каким будет ток на частоте 25,0 кГц?

7. А 20.Источник 0 кГц, 16,0 В, подключенный к катушке индуктивности, вырабатывает ток 2,00 А. Что такое индуктивность?

8. Источник 20,0 Гц, 16,0 В вырабатывает ток 2,00 мА при подключении к конденсатору. Какая емкость?

9. (a) Катушка индуктивности, предназначенная для фильтрации высокочастотного шума от энергии, подаваемой на персональный компьютер, включается последовательно с компьютером. Какая минимальная индуктивность должна обеспечивать реактивное сопротивление 2,00 кОм для шума 15,0 кГц? (б) Каково его реактивное сопротивление при 60?0 Гц?

10. Конденсатор на рисунке 4 (а) предназначен для фильтрации низкочастотных сигналов, препятствуя их передаче между цепями. (а) Какая емкость необходима для создания реактивного сопротивления 100 кОм при частоте 120 Гц? (б) Каким будет его реактивное сопротивление на частоте 1,00 МГц? (c) Обсудите значение ваших ответов на (a) и (b).

11. Конденсатор на Рисунке 4 (b) будет фильтровать высокочастотные сигналы, замыкая их на землю / землю. (a) Какая емкость необходима для получения реактивного сопротивления [латекса] \ text {10.0 м \ Omega} [/ latex] для сигнала 5,00 кГц? (б) Каким будет его реактивное сопротивление при 3,00 Гц? (c) Обсудите значение ваших ответов на (a) и (b).

12. Необоснованные результаты При регистрации напряжений, обусловленных мозговой активностью (ЭЭГ), сигнал 10,0 мВ с частотой 0,500 Гц подается на конденсатор, создавая ток 100 мА. Сопротивление незначительное. а) Какая емкость? б) Что неразумного в этом результате? (c) Какое предположение или предпосылка ответственны?

13. Создайте свою проблему Рассмотрите возможность использования индуктора последовательно с компьютером, работающим от электричества 60 Гц. Постройте задачу, в которой вы вычисляете относительное снижение напряжения входящего высокочастотного шума по сравнению с напряжением 60 Гц. Среди вещей, которые следует учитывать, — допустимое последовательное реактивное сопротивление катушки индуктивности для мощности 60 Гц и вероятные частоты шума, проходящего через линии электропередач.

Глоссарий

индуктивное реактивное сопротивление:
противодействие катушки индуктивности изменению тока; рассчитывается по X L = 2π fL
емкостное реактивное сопротивление:
сопротивление конденсатора изменению тока; рассчитывается по [latex] {X} _ {C} = \ frac {1} {2 \ pi fC} \\ [/ latex]

Избранные решения проблем и упражнения

1.531 Гц

3. 1,33 нФ

5. (а) 2,55 А (б) 1,53 мА

7. 63,7 мкГн

9. (а) 21,2 мГн (б) 8,00 Ом

Емкостное реактивное сопротивление — обзор

X

Символ реактивного сопротивления.

X C

Символ емкостного реактивного сопротивления.

X L

Символ индуктивного реактивного сопротивления.

Демодуляция X и Z

Система демодуляции цветного ТВ, в которой два повторно вставленных сигнала поднесущей 3,58 МГц различаются примерно на 60 °, а не на обычные 90 °. Напряжения R — Y, B — Y и G — Y выводятся из демодулированных сигналов, и эти напряжения управляют тремя пушками кинескопа. Важным преимуществом этой системы является то, что схема приемника проще, чем требуется при I- и Q-демодуляции.

Ось X

1.Ось отсчета в кристалле кварца. 2. Горизонтальная ось в системе прямоугольных координат. 3. Направление по горизонтали или слева направо в двумерной системе координат. X-X обозначает одно направление, в котором следует метод пошагового повторения.

Диапазон X

Радиочастотный диапазон от 5200 до 11000 МГц с длинами волн от 5,77 до 2,75 см.

X-образный стержень

Прямоугольный кристаллический стержень, обычно вырезанный из Z-образного сечения, вытянутый параллельно X, а его края параллельны X, Y и Z.

X-конденсатор

Конденсатор для подавления радиопомех, предназначенный для приложений, в которых отказ конденсатора не приведет к опасности поражения электрическим током.

Х-образный кристалл

Кристалл, вырезанный так, что его основные поверхности перпендикулярны электрической оси (X) исходного кристалла кварца.

ксенон

Инертный газ, используемый в некоторых тиратронах и других газовых трубках.

ксеноновая импульсная лампа

Источник некогерентного белого света высокой интенсивности; он работает, разряжая конденсатор через трубку с газом ксеноном. Такое устройство часто используется в качестве источника излучения накачки для различных лазеров с оптическим возбуждением.

ксерографический принтер

Устройство для печати оптического изображения на бумаге; светлые и темные области представлены электростатически заряженными и незаряженными участками на бумаге.Порошковые чернила, присыпанные пылью на бумаге, прилипают к заряженным участкам и затем плавятся в бумаге под воздействием тепла.

ксерографическая запись

Запись, полученная с помощью ксерографии.

ксерография

1. Эта ветвь электростатической электрофотографии, в которой изображения формируются на фотопроводящей изолирующей среде с помощью инфракрасного, видимого или ультрафиолетового излучения. Затем среду присыпают порошком, который прилипает только к электростатически заряженному изображению.Затем применяется тепло, чтобы сплавить порошок в постоянное изображение. 2. Процесс печати электростатической электрофотографии, в котором используется фотопроводящая изолирующая среда в сочетании с инфракрасным, видимым или ультрафиолетовым излучением для создания структур скрытого электростатического заряда для достижения наблюдаемой записи.

xeroprinting

Эта ветвь электростатической электрофотографии, в которой узор из изоляционного материала на проводящей среде используется для формирования структур электростатического заряда для использования при копировании.

xeroradiography

Процесс печати электростатической электрофотографии, в котором используется светопроводящая изолирующая среда в сочетании с рентгеновскими или гамма-лучами для создания структур скрытого электростатического заряда для получения наблюдаемого рисунка.

ксерорадиографическое оборудование

Оборудование, использующее принципы электростатики и фотопроводимости для записи рентгеновских изображений на сенсибилизированную пластину через короткое время после экспонирования.

xfmr

Сокращенное обозначение трансформатора.

xistor

Сокращенное обозначение транзистора.

Разъем XLR

Экранированный трехжильный микрофонный штекер или розетка с фиксатором для разблокировки пальцем для предотвращения случайного извлечения. Стандартный разъем для профессиональных пользователей микрофонов.

xmitter

Аббревиатура передатчика.Также сокращенно trans или xmtr.

xmsn

Сокращенное обозначение передачи.

xmtr

Аббревиатура передатчика. Также сокращенно транс или xmitter.

X-off

Передатчик выключен.

X-on

Датчик включен.

X-частица

Частица, имеющая такой же отрицательный заряд, что и электрон, но масса между электроном и протоном.Он создается космическим излучением, падающим на молекулы газа или фактически составляющим часть космических лучей.

Рентгеновский аппарат

Рентгеновская трубка и принадлежности к ней, включая рентгеновский аппарат.

Рентгеновская кристаллография

1. Использование рентгеновских лучей для изучения расположения атомов в кристалле. 2. Изучение структуры кристаллических материалов с использованием взаимодействия рентгеновских лучей и электронной плотности кристалла (дифракции).

Устройство обнаружения рентгеновских лучей

Устройство, обнаруживающее неоднородности поверхности и объема твердых тел с помощью рентгеновских лучей.

Камера для дифракции рентгеновских лучей

Камера, которая направляет пучок рентгеновских лучей на образец неизвестного материала и позволяет полученным дифрагированным лучам воздействовать на полосу пленки.

Картина дифракции рентгеновских лучей

Картина, полученная на пленке, экспонированной с помощью рентгеновской дифракционной камеры.Он состоит из частей кругов с разным расстоянием между ними в зависимости от исследуемого материала.

Рентгеновский гониометр

Прибор, который определяет положение электрических осей кристалла кварца путем отражения рентгеновских лучей от атомных плоскостей кристалла.

Рентгеновские лучи

Также называемые рентгеновскими лучами. Проникающее излучение похоже на свет, но имеет гораздо более короткие длины волн (от 10 –7 до 10 –10 см).Обычно они возникают при бомбардировке металлической мишени потоком высокоскоростных электронов.

Рентгеновский спектрограф

Инструмент, который используется для построения диаграмм дифракции рентгеновских лучей, такой как рентгеновский спектрометр с фотографическими или другими регистрирующими устройствами.

Рентгеновский спектрометр

1. Прибор для получения спектра рентгеновского излучения и измерения длин волн его компонентов. 2. Прибор, предназначенный для получения рентгеновского спектра материала в качестве помощи в его идентификации.Этот метод особенно полезен, когда материал не может быть физически разрушен.

Спектр рентгеновского излучения

Расположение пучка рентгеновских лучей в порядке длины волны.

Рентгеновский толщиномер

Бесконтактный толщиномер, используемый для измерения и индикации толщины движущегося холоднокатаного стального листа в процессе прокатки. Рентгеновский луч, направленный через лист, поглощается пропорционально толщине материала и его атомному номеру, и измерение количества поглощения дает непрерывное указание толщины листа.

Рентгеновская трубка

Вакуумная трубка, в которой рентгеновские лучи производятся путем бомбардировки мишени высокоскоростными электронами, ускоренными электростатическим полем.

Мишень для рентгеновской трубки

Также известен как антикатод. Электрод или электродная секция, на которую фокусируется электронный луч и который излучает рентгеновские лучи.

xso

Аббревиатура для генератора с кварцевой стабилизацией.

xtal

Аббревиатура кристалла.

X-wave

Один из двух компонентов, на которые магнитное поле Земли делит радиоволну в ионосфере. Другой компонент — обыкновенная, или О-, волна.

Кристалл XY-огранки

Кристалл, ограненный таким образом, что его характеристики находятся между кристаллами X- и Y-огранки.

Плоттер XY

1.Устройство, используемое вместе с компьютером для нанесения координатных точек в виде графика. 2. Компьютерное устройство вывода, которое реагирует на цифровые сигналы предварительно записанных и / или обработанных данных путем распечатки линейных сегментов. Эти данные, которые могут включать буквенно-цифровые символы, диаграммы, таблицы или рисунки, загружаются из памяти компьютера со скоростью, достаточной для работы плоттера. XY-плоттер нельзя использовать для прямой записи аналоговых сигналов без дигитайзеров.

Регистратор XY

1.Регистратор, который отслеживает на графике отношения между двумя переменными, ни одна из которых не является временем. Иногда диаграмма перемещается, и одна из переменных контролируется так, что взаимосвязь действительно увеличивается пропорционально времени. 2. Регистратор, в котором два сигнала одновременно записываются одним пером, которое приводится в движение в одном направлении (ось X) одним сигналом, а в другом направлении (ось Y) — вторым сигналом. 3. Регистратор данных, который используется для записи изменения одного параметра по отношению к другому.Например, изменение давления в зависимости от температуры. Для этих самописцев доступен широкий спектр преобразователей для преобразования физических параметров в электрические сигналы, используемые в самописце. Датчики давления, термопары, тензодатчики и акселерометры — вот несколько примеров. 4. Тип регистратора, который реагирует на поступающие аналоговые сигналы по мере их появления. Сигналы печатаются на графике заранее определенного размера, который может охватывать тестовые периоды от нескольких секунд до целого года. XY-рекордер записывает непрерывные линии.Кроме того, скорость отклика прибора важна для точности записи.

Переключатель XY

Переключатель с дистанционным управлением, расположенный так, что дворники перемещаются вперед и назад по горизонтали.

В чем разница между сопротивлением, реактивным сопротивлением и импедансом?

Цепи постоянного тока относительно легко анализировать, поскольку ток течет в одном направлении, а сопротивление является основным элементом цепи.Цепи переменного тока, с другой стороны, более сложны, поскольку напряжение и ток меняют направление с заданной частотой. В то время как цепи постоянного тока имеют сопротивление, цепи переменного тока часто имеют сопротивление и другое свойство, известное как реактивное сопротивление. Импеданс — это комбинация сопротивления и реактивного сопротивления.


Компоненты, известные как резисторы, предотвращают протекание тока — другими словами, они обладают свойством сопротивления. Резисторы присутствуют как в цепях переменного, так и в постоянном токе, и энергия, которая не может течь, отводится в виде тепла.Математически сопротивление — это просто напряжение, разделенное на ток.

R = сопротивление (Ом)

В = напряжение (вольт)

I = ток (амперы)


Реактивное сопротивление — это свойство, которое препятствует изменению тока и присутствует как в катушках индуктивности, так и в конденсаторах. Поскольку оно влияет только на , изменяя ток , реактивное сопротивление зависит от мощности переменного тока и зависит от частоты тока.Когда присутствует реактивное сопротивление, оно создает фазовый сдвиг на 90 градусов между напряжением и током, причем направление сдвига зависит от того, является ли компонент катушкой индуктивности или конденсатором.

Реактивное сопротивление, возникающее в катушке индуктивности, известно как индуктивное реактивное сопротивление . Когда присутствует индуктивное реактивное сопротивление, энергия накапливается в виде изменяющегося магнитного поля, и форма волны тока отстает от формы волны напряжения на 90 градусов. Индуктивное реактивное сопротивление возникает из-за устройств, в которых провод намотан по кругу, таких как катушки (включая линейные реакторы), дроссели и трансформаторы.

X L = индуктивное реактивное сопротивление (Ом)

f = частота (Гц)

L = индуктивность (Генри)

Реактивное сопротивление конденсатора известно как емкостное реактивное сопротивление . Емкостное реактивное сопротивление накапливает энергию в виде изменяющегося электрического поля и вызывает ток в отведении от напряжения на 90 градусов. Емкость создается, когда две проводящие пластины размещаются параллельно друг другу с небольшим расстоянием между ними, заполненные диэлектрическим материалом (изолятором).

X C = емкостное реактивное сопротивление (Ом)

C = емкость (фарады)


Импеданс — это комбинация сопротивления и реактивного сопротивления (как индуктивного, так и емкостного) и является комплексным числом, содержащим как действительную, так и мнимую части. (Действительная часть импеданса — это сопротивление, а мнимая часть — реактивное сопротивление.) Импеданс имеет как величину, так и фазу.

Z = величина импеданса (Ом) в последовательной цепи

X T = полное реактивное сопротивление (Ом) = X L — X C

θ = фаза полного сопротивления (градусы)

Импеданс и реактивное сопротивление | Electronics Club

Импеданс и реактивное сопротивление | Клуб электроники

Импеданс | Реактивное сопротивление | Входное сопротивление | Выходное сопротивление | Импеданс делителя напряжения

Следующая страница: Аналоговые и цифровые

См. Также: Емкость | Сопротивление

Импеданс

Импеданс (символ Z) — это мера общего сопротивления цепи току, другими словами: насколько схема препятствует потоку заряда.Это похоже на сопротивление, но также учитывает влияние емкости и индуктивности. Импеданс измеряется в омах ().

Импеданс сложнее, чем сопротивление, потому что влияние емкости и индуктивность зависит от частоты тока, проходящего через цепь, и это означает, что импеданс изменяется в зависимости от частоты . Эффект сопротивления постоянен независимо от частоты.

В = напряжение в вольтах (В)
I = ток в амперах (А)
Z = полное сопротивление в Ом ()
R = сопротивление в Ом ()

Импеданс в простых цепях

Термин «импеданс» часто используется (совершенно правильно) для простых цепей. которые не имеют емкости или индуктивности, например для обозначения их «входное сопротивление» или «выходное сопротивление».Сначала это может показаться запутанным, но для этих простых схем вы можете предположить, что это просто другое слово для обозначения сопротивления.

Импеданс можно разделить на две части:
  • Сопротивление R (часть, которая постоянна независимо от частоты)
  • Реактивное сопротивление X (часть, которая зависит от частоты из-за емкости и индуктивности)

Емкость и индуктивность вызывают сдвиг фазы (см. Примечание) между ток и напряжение, что означает, что сопротивление и реактивное сопротивление нельзя просто сложить для получения полного сопротивления.Вместо этого они должны быть добавлены как векторы с реактивным сопротивлением, перпендикулярным сопротивлению, как показано на диаграмме.

Четыре электрические величины определяют полное сопротивление (Z) цепи: сопротивление (R), емкость (C), индуктивность (L) и частота (f).

В следующем разделе, посвященном реактивному сопротивлению, объясняется, как емкость, индуктивность и частота влияют на импеданс.

Что означает «фазовый сдвиг»?

Фазовый сдвиг означает, что ток и напряжение не совпадают друг с другом.Подумайте о зарядке конденсатора. Когда напряжение на конденсаторе равно нулю, ток максимален; когда конденсатор заряжен и напряжение максимальное, ток минимальный. Зарядка и разрядка происходят постоянно с переменным током, и ток вскоре достигает максимума. до того, как напряжение достигнет своего максимума: мы говорим, что ток опережает напряжение.


Реактивное сопротивление, X

Реактивное сопротивление (символ X) — это мера противостояния емкости и индуктивности. к текущему.Реактивное сопротивление зависит от частоты электрического сигнала. Реактивное сопротивление измеряется в омах ().

Существует два типа реактивного сопротивления: емкостное реактивное сопротивление (Xc) и индуктивное реактивное сопротивление (X L ).

Полное реактивное сопротивление (X) — это разница между двумя:

Полное реактивное сопротивление, X = X L — Xc

Емкостное реактивное сопротивление Xc

Емкостное реактивное сопротивление (Xc) велико на низких частотах и ​​мало на высоких частотах.Для постоянного постоянного тока, который является нулевой частотой (f = 0 Гц), Xc бесконечно (полное противодействие), Это означает, что конденсаторы пропускают переменный ток, но блокируют постоянный ток .

Емкостное реактивное сопротивление, Xc = 1
2fC

Xc = реактивное сопротивление в Ом ()
f = частота в герцах (Гц)
C = емкость в фарадах (Ф)

Например: конденсатор 1 мкФ имеет реактивное сопротивление 3.2k для сигнала 50 Гц, но когда частота выше 10 кГц, его реактивное сопротивление составляет всего 16.

Индуктивное реактивное сопротивление, X

L

Индуктивное реактивное сопротивление, X L мало на низких частотах и ​​большое на высоких. Для стабильного постоянного тока (нулевая частота) X L равно нулю (нет противодействия), Это означает, что катушки индуктивности пропускают постоянный ток, но блокируют высокочастотный переменный ток .

Индуктивное реактивное сопротивление, X L = 2fL

X L = реактивное сопротивление в Ом ()
f = частота в герцах (Гц)
L = индуктивность в генри (H)

Например: катушка индуктивности 1 мГн имеет реактивное сопротивление всего 0.3 для сигнала 50 Гц, но когда частота выше 10 кГц, его реактивное сопротивление равно 63.



Входное сопротивление Z

IN

Входной импеданс (Z IN ) — это импеданс, «видимый» всем, что подключено к входу. схемы или устройства (например, усилителя). Это совокупный эффект всего сопротивления, емкость и индуктивность, подключенные к входу внутри схемы или устройства.

Термин «входной импеданс» является нормальным даже в простых случаях, когда имеется только сопротивление. вместо этого можно использовать термин «входное сопротивление».На самом деле обычно разумно предположить что входное сопротивление — это просто сопротивление, при условии, что входной сигнал имеет низкую частоту (менее 1 кГц).

Влияние емкости и индуктивности зависит от частоты, поэтому, если они присутствуют, входное сопротивление будет меняться в зависимости от частоты. Влияние емкости и индуктивности обычно наиболее значимо на высоких частотах.

Обычно входной импеданс должен быть высоким , как минимум в десять раз превышающим выходной импеданс. схемы (или компонента), подающей сигнал на вход.Это гарантирует, что вход не будет «перегружен». источник сигнала и значительно уменьшите силу (напряжение) сигнала.


Выходное сопротивление Z

OUT

Выход любой схемы или устройства эквивалентен выходному сопротивлению (Z OUT ) последовательно с идеальным источником напряжения (В ИСТОЧНИК ). Это называется эквивалентная схема и представляет собой совокупное влияние всех источников напряжения, сопротивления, емкость и индуктивность, подключенные к выходу внутри схемы или устройства.Обратите внимание, что V SOURCE обычно , а не , как напряжение питания Vs.

Термин «выходной импеданс» является нормальным даже в простых случаях, когда имеется только сопротивление. вместо этого можно использовать термин «выходное сопротивление». На самом деле обычно разумно предположить что выходное сопротивление — это просто сопротивление, при условии, что выходной сигнал имеет низкую частоту (менее 1 кГц).

Схема замещения любого выхода

Влияние емкости и индуктивности зависит от частоты, поэтому, если они присутствуют, выходное сопротивление будет меняться в зависимости от частоты.Влияние емкости и индуктивности обычно наиболее значимо на высоких частотах.

Обычно выходное сопротивление должно быть низким , менее одной десятой полного сопротивления нагрузки подключен к выходу. Если выходной импеданс слишком высок, он не сможет обеспечить достаточно сильный сигнал к нагрузке, потому что большая часть напряжения сигнала будет « потеряна » внутри цепи, управляющей током через выходное сопротивление Z OUT . Нагрузка может быть отдельным компонентом или входным сопротивлением другой цепи.

Низкое выходное сопротивление , Z ВЫХ << Z НАГРУЗКА
Большая часть В ИСТОЧНИК появляется на нагрузке, очень небольшое напряжение «теряется» управляя выходным током через выходное сопротивление. Обычно это лучшая аранжировка.

Согласованные импедансы , Z ВЫХ = Z НАГРУЗКА
Половина V ИСТОЧНИК отображается в нагрузке, другая половина «потеряна» управляя выходным током через выходное сопротивление.Такое расположение полезно в некоторых ситуации (например, усилитель, управляющий громкоговорителем), потому что он обеспечивает максимальную мощность нагрузка . Обратите внимание, что равное количество энергии тратится впустую, управляя выходным током через Z OUT , КПД 50%.

Высокое выходное сопротивление , Z ВЫХ >> Z НАГРУЗКА
Лишь небольшая часть V SOURCE отображается в нагрузке, большая часть «потеряна» управляя выходным током через выходное сопротивление.Такое расположение неудовлетворительно.

Нагрузка может быть однокомпонентной или
входным сопротивлением другой цепи



Выходное сопротивление делителя напряжения

Делители напряжения

широко используются в электронике, например, для подключения входного преобразователя, такого как LDR, к входу схемы.

Для успешного использования выходное сопротивление делителя напряжения должно быть намного меньше. чем входное сопротивление подключенной к нему цепи.

В идеале выходное сопротивление должно быть меньше одной десятой входного сопротивления.

В эквивалентной схеме делителя напряжения выходное сопротивление — это просто сопротивление. и можно использовать термин «выходное сопротивление». R OUT равно к двум параллельно подключенным сопротивлениям (R1 и R2):

Выходное сопротивление, R OUT = R1 × R2
R1 + R2

Источник напряжения V ИСТОЧНИК в эквивалентной схеме значение выходное напряжение Vo, когда к выходу ничего не подключено (и, следовательно, нет выходного тока).Иногда его называют напряжением холостого хода.

Источник напряжения, В ИСТОЧНИК = Vs × R2
R1 + R2

Схема замещения делителя напряжения

Делитель напряжения с LDR


Следующая страница: Аналоговый и цифровой | Исследование


Политика конфиденциальности и файлы cookie

Этот сайт не собирает личную информацию.Если вы отправите электронное письмо, ваш адрес электронной почты и любая личная информация будет используется только для ответа на ваше сообщение, оно не будет передано никому. На этом веб-сайте отображается реклама, если вы нажмете на рекламодатель может знать, что вы пришли с этого сайта, и я могу быть вознагражден. Рекламодателям не передается никакая личная информация. Этот веб-сайт использует некоторые файлы cookie, которые классифицируются как «строго необходимые», они необходимы для работы веб-сайта и не могут быть отклонены, но они не содержат никакой личной информации.Этот веб-сайт использует службу Google AdSense, которая использует файлы cookie для показа рекламы на основе использования вами веб-сайтов. (включая этот), как объяснил Google. Чтобы узнать, как удалить файлы cookie и управлять ими в своем браузере, пожалуйста, посетите AboutCookies.org.

electronicsclub.info © Джон Хьюс 2021 г.

онлайн-курсов PDH. PDH для профессиональных инженеров. ПДХ Инжиниринг.

«Мне нравится широта ваших курсов по HVAC; не только экологичность или экономия энергии

курсов.»

Russell Bailey, P.E.

Нью-Йорк

«Это укрепило мои текущие знания и научило меня еще нескольким новым вещам.

, чтобы познакомить меня с новыми источниками

информации.

Стивен Дедак, П.Е.

Нью-Джерси

«Материал был очень информативным и организованным.Я многому научился и они были

очень быстро отвечает на вопросы.

Это было на высшем уровне. Будет использовать

снова. Спасибо. «

Blair Hayward, P.E.

Альберта, Канада

«Простой в использовании веб-сайт. Хорошо организованный. Я действительно воспользуюсь вашими услугами снова.

проеду по вашей компании

имя другим на работе.»

Roy Pfleiderer, P.E.

Нью-Йорк

«Справочные материалы были превосходными, и курс был очень информативным, особенно потому, что я думал, что я уже знаком с

с подробной информацией о Канзасе

Городская авария Хаятт. «

Майкл Морган, P.E.

Техас

«Мне очень нравится ваша бизнес-модель.Мне нравится просматривать текст перед покупкой. Нашел класс

информативно и полезно

в моей работе ».

Вильям Сенкевич, П.Е.

Флорида

«У вас большой выбор курсов, а статьи очень информативны. You

— лучшее, что я нашел ».

Рассел Смит, П.E.

Пенсильвания

«Я считаю, что такой подход позволяет работающему инженеру легко зарабатывать PDH, давая время на просмотр

материал «

Jesus Sierra, P.E.

Калифорния

«Спасибо, что позволили мне просмотреть неправильные ответы. На самом деле это

человек узнает больше

от сбоев.»

John Scondras, P.E.

Пенсильвания

«Курс составлен хорошо, и использование тематических исследований является эффективным.

способ обучения »

Джек Лундберг, P.E.

Висконсин

«Я очень впечатлен тем, как вы представляете курсы, т.е. позволяете

студент для ознакомления с курсом

материалов до оплаты и

получает викторину.»

Арвин Свангер, П.Е.

Вирджиния

«Спасибо за то, что вы предложили все эти замечательные курсы. Я определенно выучил и

получил огромное удовольствие «.

Мехди Рахими, П.Е.

Нью-Йорк

«Я очень доволен предлагаемыми курсами, качеством материалов и простотой поиска.

на связи

курсов.»

Уильям Валериоти, P.E.

Техас

«Этот материал в значительной степени оправдал мои ожидания. По курсу было легко следовать. Фотографии в основном обеспечивали хорошее наглядное представление о

.

обсуждаемых тем ».

Майкл Райан, P.E.

Пенсильвания

«Именно то, что я искал. Потребовался 1 балл по этике, и я нашел его здесь.»

Джеральд Нотт, П.Е.

Нью-Джерси

«Это был мой первый онлайн-опыт получения необходимых мне кредитов PDH. Это было

информативно, выгодно и экономично.

Я очень рекомендую

всем инженерам. »

Джеймс Шурелл, P.E.

Огайо

«Я понимаю, что вопросы относятся к« реальному миру »и имеют отношение к моей практике, и

не на основе какой-то неясной раздел

законов, которые не применяются

«нормальная» практика.»

Марк Каноник, П.Е.

Нью-Йорк

«Отличный опыт! Я многому научился, чтобы перенести его на свой медицинский прибор.

организация «

Иван Харлан, П.Е.

Теннесси

«Материалы курса имели хорошее содержание, не слишком математическое, с хорошим акцентом на практическое применение технологий».

Юджин Бойл, П.E.

Калифорния

«Это был очень приятный опыт. Тема была интересной и хорошо изложенной,

а онлайн-формат был очень

Доступно и просто

использовать. Большое спасибо. «

Патрисия Адамс, P.E.

Канзас

«Отличный способ добиться соответствия требованиям PE Continuing Education в рамках ограничений по времени лицензиата.»

Joseph Frissora, P.E.

Нью-Джерси

«Должен признаться, я действительно многому научился. Помогает иметь распечатанный тест во время

обзор текстового материала. Я

также оценил просмотр

фактических случаев предоставлено.

Жаклин Брукс, П.Е.

Флорида

«Документ» Общие ошибки ADA при проектировании объектов «очень полезен.Модель

испытание потребовало исследования в

документ но ответы были

в наличии «

Гарольд Катлер, П.Е.

Массачусетс

«Я эффективно использовал свое время. Спасибо за широкий выбор вариантов.

в транспортной инженерии, что мне нужно

для выполнения требований

Сертификат ВОМ.»

Джозеф Гилрой, П.Е.

Иллинойс

«Очень удобный и доступный способ заработать CEU для моих требований PG в Делавэре».

Ричард Роадс, P.E.

Мэриленд

«Я многому научился с защитным заземлением. Пока все курсы, которые я прошел, были отличными.

Надеюсь увидеть больше 40%

курсов со скидкой.»

Кристина Николас, П.Е.

Нью-Йорк

«Только что сдал экзамен по радиологическим стандартам и с нетерпением жду возможности сдать дополнительный

курсов. Процесс прост, и

намного эффективнее, чем

в пути «.

Деннис Мейер, P.E.

Айдахо

«Услуги, предоставляемые CEDengineering, очень полезны для Professional

Инженеры получат блоки PDH

в любое время.Очень удобно ».

Пол Абелла, P.E.

Аризона

«Пока все отлично! Поскольку я постоянно работаю матерью двоих детей, у меня мало

время искать где

получить мои кредиты от. «

Кристен Фаррелл, P.E.

Висконсин

«Это было очень познавательно и познавательно.Легко для понимания с иллюстрациями

и графики; определенно делает это

легче поглотить все

теорий. «

Виктор Окампо, P.Eng.

Альберта, Канада

«Хороший обзор принципов работы с полупроводниками. Мне понравилось пройти курс по

.

мой собственный темп во время моего утром

до метро

на работу.»

Клиффорд Гринблатт, П.Е.

Мэриленд

«Просто найти интересные курсы, скачать документы и взять

викторина. Я бы очень рекомендовал

вам на любой PE нужно

CE единиц. «

Марк Хардкасл, П.Е.

Миссури

«Очень хороший выбор тем из многих областей техники.»

Randall Dreiling, P.E.

Миссури

«Я заново узнал то, что забыл. Я также рад оказать финансовую помощь

по ваш промо-адрес который

сниженная цена

на 40% «

Конрадо Казем, П.E.

Теннесси

«Отличный курс по разумной цене. Воспользуюсь вашими услугами в будущем».

Charles Fleischer, P.E.

Нью-Йорк

«Это был хороший тест и фактически подтвердил, что я прочитал профессиональную этику

кодов и Нью-Мексико

правил. «

Брун Гильберт, П.E.

Калифорния

«Мне очень понравились занятия. Они стоили потраченного времени и усилий».

Дэвид Рейнольдс, P.E.

Канзас

«Очень доволен качеством тестовых документов. Буду использовать CEDengineerng

.

при необходимости дополнительно

сертификация. «

Томас Каппеллин, П.E.

Иллинойс

«У меня истек срок действия курса, но вы все же выполнили свое обязательство и дали

мне то, за что я заплатил — много

оценено! «

Джефф Ханслик, P.E.

Оклахома

«CEDengineering предлагает удобные, экономичные и актуальные курсы.

для инженера »

Майк Зайдл, П.E.

Небраска

«Курс был по разумной цене, материал был кратким, а

хорошо организовано. «

Glen Schwartz, P.E.

Нью-Джерси

«Вопросы подходили для уроков, а материал урока —

.

хороший справочный материал

для деревянного дизайна. «

Брайан Адамс, П.E.

Миннесота

«Отлично, я смог получить полезные рекомендации по простому телефонному звонку.»

Роберт Велнер, P.E.

Нью-Йорк

«У меня был большой опыт работы в прибрежном строительстве — проектирование

Building курс и

очень рекомендую

Денис Солано, P.E.

Флорида

«Очень понятный, хорошо организованный веб-сайт. Материалы курса этики Нью-Джерси были очень хорошими

хорошо подготовлены. »

Юджин Брэкбилл, P.E.

Коннектикут

«Очень хороший опыт. Мне нравится возможность загружать учебные материалы на

.

обзор везде и

всякий раз, когда.»

Тим Чиддикс, P.E.

Колорадо

«Отлично! Поддерживаю широкий выбор тем на выбор».

Уильям Бараттино, P.E.

Вирджиния

«Процесс прямой, без всякой ерунды. Хороший опыт».

Тайрон Бааш, П.E.

Иллинойс

«Вопросы на экзамене были зондирующими и продемонстрировали понимание

материала. Полная

и комплексное. »

Майкл Тобин, P.E.

Аризона

«Это мой второй курс, и мне понравилось то, что мне предлагали курс

поможет по моей линии

работ.»

Рики Хефлин, P.E.

Оклахома

«Очень быстро и легко ориентироваться. Я определенно буду использовать этот сайт снова».

Анджела Уотсон, П.Е.

Монтана

«Легко выполнить. Никакой путаницы при подходе к сдаче теста или записи сертификата».

Кеннет Пейдж, П.E.

Мэриленд

«Это был отличный источник информации о солнечном нагреве воды. Информативный

и отличный освежитель ».

Луан Мане, П.Е.

Conneticut

«Мне нравится подход, когда я подписываюсь и могу читать материалы в автономном режиме, а затем

Вернуться, чтобы пройти викторину «

Алекс Млсна, П.E.

Индиана

«Я оценил объем информации, предоставленной для класса. Я знаю

это вся информация, которую я могу

использование в реальных жизненных ситуациях »

Натали Дерингер, P.E.

Южная Дакота

«Обзорные материалы и образец теста были достаточно подробными, чтобы позволить мне

успешно завершено

курс.»

Ира Бродский, П.Е.

Нью-Джерси

«Веб-сайт прост в использовании, вы можете скачать материал для изучения, а потом вернуться

и пройдите викторину. Очень

удобно и на моем

собственный график «

Майкл Гладд, P.E.

Грузия

«Спасибо за хорошие курсы на протяжении многих лет.»

Деннис Фундзак, П.Е.

Огайо

«Очень легко зарегистрироваться, получить доступ к курсу, пройти тест и распечатать PDH

сертификат. Спасибо за создание

процесс простой. »

Фред Шейбе, P.E.

Висконсин

«Положительный опыт.Быстро нашел курс, который соответствовал моим потребностям, и закончил

один час PDH в

один час. «

Стив Торкильдсон, P.E.

Южная Каролина

«Мне понравилась возможность скачать документы для проверки содержания

и пригодность, до

имея заплатить за

материал

Ричард Вимеленберг, P.E.

Мэриленд

«Это хорошее напоминание об ЭЭ для инженеров, не занимающихся электричеством».

Дуглас Стаффорд, П.Е.

Техас

«Всегда есть возможности для улучшения, но я ничего не могу придумать в вашем

.

процесс, который требует

улучшение.»

Thomas Stalcup, P.E.

Арканзас

«Мне очень нравится удобство участия в онлайн-викторине и получение сразу

сертификат. «

Марлен Делани, П.Е.

Иллинойс

«Учебные модули CEDengineering — это очень удобный способ доступа к информации по номеру

.

много разные технические зоны за пределами

своя специализация без

приходится путешествовать.2 = -1 \ $. Это все, что вам нужно для этих расчетов. Причина: получение сложного корня многозначно, но возведение в квадрат не вызывает сомнений. Так что избегайте получения корня, если вы можете сделать это с помощью возведения в квадрат.

И да, я определенно предпочитаю рассматривать реактивное сопротивление конденсатора \ $ C \ $ как отрицательное, чтобы выразить разность фаз между током и напряжением, по сравнению с теми же самыми вещами в / на катушке индуктивности.

На мой взгляд, даже лучше различать величину и значение реактивного сопротивления: используйте символ каретки, чтобы различать их, как мы уже делаем для напряжения или тока: \ $ V \ $ и \ $ \ hat V \ $ и \ $ i \ $ и \ $ \ hat i \ $.Эти специальные символы трудно увидеть в режиме обычного текста, но с этим специальным форматом, дружественным к математике, он действительно выглядит красиво.

Я предлагаю сделать то же самое с \ $ X \ $, поэтому для конденсатора \ $ C \ $ определите \ $ X = — \ frac {1} {\ omega C} \ $ и \ $ | X | = \ hat X = \ frac {1} {\ omega C} \ $, и с этого момента, когда вы хотите определить величину реактивного сопротивления, используйте \ $ \ hat X \ $. 2} $$ Обратите внимание, что чувствительность \ $ Y \ $, очевидно, будет иметь положительное значение, если реактивное сопротивление \ $ X <0 \ $.2} = \ omega C $$ что, как и ожидалось, является положительным числом: \ $ Y> 0 \ $

Обратите внимание, что для конденсатора \ $ C \ $ реактивное сопротивление \ $ X = — \ frac {1} {Y} \ $, где \ $ Y \ $ = восприимчивость \ $ C \ $.

Также обратите внимание, что изменение знака означает, что фаза тоже изменилась, и это так, как должно быть: потому что на конденсаторе его напряжение на нем на 90 градусов отстает от проходящего через него тока.

Если вы посмотрите на реактивное сопротивление («сопротивление переменному току») конденсатора) \ $ \ frac {V_C} {I_C} = Z_C \ $, вы должны получить отрицательный знак, отражающий то, что напряжение отстает от тока и что означает, что реактивное сопротивление \ $ X \ $ конденсатора \ $ C \ $ должно иметь отрицательный знак.

Глядя на \ $ \ frac {I_C} {V_C} = Y_C \ $, вы смотрите на ток относительно напряжения, и поскольку ток на 90 градусов опережает напряжение, проводимость («проводимость по переменному току») конденсатор \ $ Y_C \ $ должен быть положительным.

Емкостное реактивное сопротивление

— Как найти последовательные и параллельные конденсаторы

Найдите емкостное реактивное сопротивление цепи с последовательными или параллельными конденсаторами, используя этот простой двухэтапный процесс.

Шаг 1: Найдите общую емкость цепи

Предположим, у нас есть три конденсатора, 12 Ф, 20 Ф и 30 Ф, подключенных к источнику с частотой 60 Гц. Каково полное емкостное реактивное сопротивление (X C ) при последовательном или параллельном подключении?

1А. Для конденсаторов серии

Когда конденсаторы соединены последовательно, общая емкость меньше, чем любая из отдельных емкостей последовательных конденсаторов.Если два или более конденсатора соединены последовательно, общий эффект будет таким, как у одного (эквивалентного) конденсатора, имеющего суммарное расстояние между пластинами отдельных конденсаторов.

Конденсаторы серии Пример:

1/12 = 0,083, 1/20 = 0,050, 1/30 = 0,033

0,083 + 0,050 + 0,033 = 0,166

1 / 0,163 = 6,02 мкФ

Примечание: математика упрощена для целей иллюстрации. Для более точных чисел воспользуйтесь калькулятором.

1Б. Для параллельных конденсаторов

При параллельном подключении конденсаторов общая емкость складывается из емкостей отдельных конденсаторов. Если два или более конденсатора соединены параллельно, общий эффект будет таким, как у одного эквивалентного конденсатора, имеющего сумму площадей пластин отдельных конденсаторов.

Параллельные конденсаторы Пример:

12 + 20 + 30 = 62 мкФ


Шаг 2: Найдите емкостное сопротивление

Как и сопротивление, реактивное сопротивление измеряется в Ом, но ему присваивается символ X, чтобы отличить его от чисто резистивного значения R, и, поскольку рассматриваемый компонент является конденсатором, реактивное сопротивление конденсатора называется емкостным реактивным сопротивлением (X C ) который измеряется в Ом.

Поскольку конденсаторы заряжаются и разряжаются пропорционально скорости изменения напряжения на них, чем быстрее изменяется напряжение, тем больше тока протекает. Точно так же, чем медленнее изменяется напряжение, тем меньше будет протекать ток. Это означает, что реактивное сопротивление конденсатора переменного тока «обратно пропорционально» частоте источника питания.

X C — емкостное реактивное сопротивление в омах, f — частота в герцах, а C — емкость переменного тока в фарадах. Очень важно преобразовать наш пример из микрофарад в фарады, чтобы получить правильный результат!

1 мкФ = 0,000001 F

Серия 60 Гц Пример:

6,02 мкФ = 0,000006 Ф (упрощенно)
2 х 3,14 х 60 х 0,000006 = 0,0022608
1 / 0,0022608 = 442,32 Ом

60 Гц Параллельный Пример:

62 мкФ = 0,000062 F
2 х 3.14 х 60 х 0,000062 = 0,0233616
1 / 0,0233616 = 42,805 Ом

Теперь посмотрим, что произойдет при изменении частоты на 400 Гц :

Серия 400 Гц Пример:

2 х 3,14 х 400 х 0,000006 = 0,015072
1 / 0,015072 = 66,34 Ом

Параллельный 400 Гц Пример:

2 х 3,14 х 400 х 0,000062 = 0,155744
1 / 0,155744 = 6,42 Ом


Полезные ссылки

на комментарий.

alexxlab

Добавить комментарий

Ваш адрес email не будет опубликован. Обязательные поля помечены *